You are on page 1of 41

VISIONIAS

www.visionias.in
ANSWERS & EXPLANATIONS
GENERAL STUDIES (P) TEST – 3196 (2021)

Q 1.C
• The Government has revamped the Municipal Solid Wastes (Management and Handling) Rules 2000 and
notified the new Solid Waste Management Rules, 2016 on April 8, 2016. The salient features of the SWM
Rules, 2016 are as under;
o The Rules are now applicable beyond Municipal areas and extend to urban agglomerations,
census towns, notified industrial townships, areas under the control of Indian Railways,
airports, airbase, Port and harbour, defence establishments, special economic zones, State and
Central government organizations, places of pilgrims, religious & historical importance.
o The source segregation of waste has been mandated to channelize the waste to wealth by recovery,
reuse and recycle.
o Responsibilities of Generators have been introduced to segregate waste into three streams, Wet
(Biodegradable), Dry (Plastic, Paper, metal, wood, etc.) and domestic hazardous wastes (diapers,
napkins, empty containers of cleaning agents, mosquito repellents, etc.) and handover segregated
wastes to authorized rag-pickers or waste collectors or local bodies.
o Integration of waste pickers/ rag pickers and waste dealers/ Kabadiwalas in the formal system should
be done by State Governments, and Self Help Group, or any other group to be formed.
o The generator will have to pay ‘User Fee’ to waste collector and for ‘Spot Fine’ for Littering
and Non-segregation. Hence option (c) is the correct answer.
o The landfill site shall be 100 metres away from a river, 200 metres from a pond, 500, 200 metres
away from highways, habitations, public parks and water supply wells and 20 km away from
airports/airbase.
o Local Authority/Panchayats shall prepare SWM plan with a timeline and its implementation,
segregate, adopt 3-Rs, material recovery, processing/ disposal of Waste, user fee and levy spot
fine.

Q 2.C
• National Biodiversity Authority, established in 2003 to implement the Biological Diversity Act,
2002, is an autonomous and statutory body to perform regulatory and advisory functions for
Government of India on issues of conservation of biodiversity, sustainable use of its components and
realization of fair and equitable sharing of benefits arising out of the use of biological resources.
• The Biological Diversity Act (2002) mandates the implementation of the provisions of the Act through the
decentralized system with the NBA focusing on advice the Central Government on matters relating to the
conservation of biodiversity, sustainable use of its components and equitable sharing of benefits arising
out of the utilization of biological resources.
• As part of its mandatory functions, NBA regulates access to India’s biological resources and
associated knowledge by Indians and non‐Indians as well, for various kinds of activities, such as
access for research, commercial utilization and obtaining of IPRs for the inventions based on any research
or information on a biological resource obtained from India.
• The State Biodiversity Board (SBBs) focus on advice the State Government, subject to any guidelines
issued by the Central Government, on matters relating to the conservation of biodiversity, sustainable use
of its components and equitable sharing of the benefits arising out of the utilization of biological
resources.
1 www.visionias.in ©Vision IAS
• There is no overlap in the functions of NBA and SBBs on issues of ABS. Their domains and functions are
very distinct from each other. All matters relating to requests by foreign individuals, companies or
institutions and all matters relating to the transfer of results of research to any foreigner, approvals for
intellectual property protection where biological resources and associated knowledge are involved will be
dealt with by NBA. All matters relating to access by Indians for commercial purposes will be under the
purview of the concerned State Biodiversity Boards. However, the benefit-sharing guidelines are to be
issued by the NBA.
• Hence both the statements are correct.

Q 3.C
• Radioactive waste is generated by nuclear power plants. It is often either heavy water or spent nuclear
fuel. The radio-active waste is highly hazardous to the environment and all life form, due to its ability to
remain in the environment for a long time and to affect the genes or the genetic material of life forms,
thereby affecting future generations.
• India generates around four tonnes of nuclear waste per gigawatt (GW) annually. Any radioactive waste
generated during the treatment processes of reprocessing is disposed following the Atomic Energy (Safe
Disposal of Radioactive Wastes) Rules 1987, promulgated under the Atomic Energy Act, 1962. Hence
statement 1 is not correct.
• Extended Producer Responsibility (EPR) is a policy approach under which producers are given a
significant responsibility – financial and/or physical – for the treatment or disposal of post-consumer
products. The Government has notified Plastic Waste Management Rules, 2016. The rules mandate the
producers, importers and brand owners to work out modalities for waste collection system based on the
principle of Extended Producer Responsibility. The EPR provision is also included in the E-waste
(Management) Rules, 2016. Hence statement 2 is correct.
• Biomedical waste comprises human & animal anatomical waste, treatment apparatus like needles,
syringes and other materials used in health care facilities in the process of treatment and research. The
Biomedical waste (BMW) management rules have been notified to efficiently manage the generated bio
waste in the country.
• Salient features of BMW Management Rules, 2016:
o The ambit of the rules has been expanded to include vaccination camps, blood donation camps,
surgical camps or any other healthcare activity.
o Establish a Bar-Code System for bags or containers containing bio-medical waste for disposal.
o Bio-medical waste is required to be segregated in 4 color coded waste categories instead of 10 to
improve the segregation of waste at source. Hence statement 3 is correct.
o Inclusion of emissions limits for Dioxin and furans.
o State Government to provide land for setting up common bio-medical waste treatment and disposal
facility.
• Hence option (c) is the correct answer.

Q 4.D
• Bureau of Energy Efficiency had launched the Energy Conservation Building Code (ECBC) 2007 to
establish minimum energy performance standards for buildings in India. Buildings consume a significant
proportion of our energy resources and the ECBC is an essential regulatory tool to curb their energy
footprint.
• The Code is applicable to buildings or building complexes that have a connected load of 100 kW or
greater or a contract demand of 120 kVA or greater and are intended to be used for commercial
purposes. Buildings intended for private residential purposes only are not covered by the Code.
Hence the correct answer is option (d).
• The provisions of this code apply to:(a) Building envelope,(b) Mechanical systems and equipment,
including heating, ventilating, and air conditioning, service hot water heating,(c) Interior and exterior
lighting, and(d) Electrical power and motors, and renewable energy systems.
• The provisions of this code do not apply to plug loads, and equipment and parts of buildings that use
energy for manufacturing processes, unless otherwise specified in the Code.
• The National Building Code of India 2016 (NBC) is the reference standard for lighting levels, heating,
ventilating, and air conditioning (HVAC), thermal comfort conditions, natural ventilation, and any other
building materials and system design criteria addressed in this Code.

2 www.visionias.in ©Vision IAS


• To comply with the Code, buildings shall
• have an Energy Performance Index Ratio (EPI Ratio) that is less than or equal to 1
• The Energy Performance Index (EPI) of a building is its annual energy consumption in kilowatt-hours
per square meter of the building. While calculating the EPI of a building, the area of unconditioned
basements shall not be included.

Q 5.B
• The Chief Wildlife Warden (CWLW) is the statutory authority, under the Wildlife (Protection) Act, 1972
who heads the Wildlife Wing of the department and exercises complete administrative control over
Protected Areas (PAs) within a state. He is the member-secretary of the State Board for Wild Life. Hence
statement 1 is correct and statement 2 is not correct.
• Statement 3 is correct: According to the Wildlife (Protection) Act, 1972 the Chief Wild Life Warden
may, if he is satisfied that any wild animal specified in Schedule I has become dangerous to human life or
is so disabled or diseased as to be beyond recovery, by Order in writing and stating the reasons therefor,
permit any person to hunt such animal or cause such animal to be hunted.

Q 6.C
• The Montreal Protocol on Substances that Deplete the Ozone Layer is the landmark multilateral
environmental agreement that regulates the production and consumption of nearly 100 man-made
chemicals referred to as ozone-depleting substances (ODS).
• When released into the atmosphere, those chemicals damage the stratospheric ozone layer, Earth’s
protective shield that protects humans and the environment from harmful levels of ultraviolet radiation
from the sun.
• Adopted on 15 September 1987, the Protocol is to date the only UN treaty ever that has been
ratified by every country on Earth - all 197 UN Member States.
• The Montreal Protocol phases down the consumption and production of the different ODS in a step-wise
manner, with different timetables for developed and developing countries (referred to as “Article 5
countries”).
• Under this treaty, all parties have specific responsibilities related to the phase-out of the different groups
of ODS, control of ODS trade, annual reporting of data, national licensing systems to control ODS
imports and exports, and other matters. Developing and developed countries have equal but differentiated
responsibilities, but most importantly, both groups of countries have binding, time-targeted and
measurable commitments.
• The substances controlled by the treaty are listed in Annexes A (CFCs, halons), B (other fully
halogenated CFCs, carbon tetrachloride, methyl chloroform), C (HCFCs), E (methyl bromide) and
F (HFCs). Nitrous oxide is not regulated by the Montreal Protocol. Hence the correct answer is
option (c).

Q 7.B
• Recent Context - One of the focuses of the new Science, Technology and Innovation Policy, currently
being drafted by the Department of Science and Technology (DST) is to increase the participation of
women in science. To this end, it will adopt a strategy from Athena Swan Charter.
• About Athena Swan Charter:
• It is a charter of the program called Athena SWAN (Scientific Women Academic Network) started
by the UK in 2005.
• The Athena SWAN Charter is an evaluation and accreditation program in the UK enhancing
gender equity in science, technology, engineering, mathematics, and medicine (STEMM).
Participating research organizations and academic institutions are required to analyze data on gender
equity and develop action plans for improvement. The program recognizes such efforts with bronze,
silver, or gold accreditation. Hence option (b) is the correct answer.
• Institutions that sign up commit to addressing unequal gender representation; tackling the gender pay gap;
removing the obstacles faced by women in career development and progression; discriminatory treatment
often experienced by trans people; gender balance of committees and zero tolerance for bullying and
sexual harassment.
• The DST will incorporate a system of grading institutes depending on the enrolment of women and the
advancement of the careers of women faculty and scientists. It will be called GATI (Gender Advancement
through Transforming Institutions). The DST has tied up with the National Assessment and Accreditation
Council, under the UGC, aiming to push gender equity through them.

3 www.visionias.in ©Vision IAS


Q 8.D
• There are two types of smog: Smog is a type of intense air pollution.
o Classical smog occurs in cool humid climate. It is a mixture of smoke, fog and sulphur dioxide.
Chemically it is a reducing mixture and so it is also called as reducing smog.
o Photochemical smog occurs in warm, dry and sunny climate. The main components of the
photochemical smog result from the action of sunlight on unsaturated hydrocarbons and nitrogen
oxides produced by automobiles and factories. Photochemical smog has high concentration of
oxidising agents and is, therefore, called as oxidising smog.
• Formation of Photochemical smog:
o Nitric oxide (NO) and nitrogen dioxide (NO2) are emitted from the combustion of fossil fuels,
along with being naturally emitted from things such as volcanos and forest fires. When exposed
to ultraviolet radiation, NO2 goes through a complex series of reactions with hydrocarbons to
produce the components of photochemical smog.
• Effects of Photochemical smog:
o The common components of photochemical smog are ozone, nitric oxide, acrolein, formaldehyde
and peroxyacetyl nitrate (PAN). Photochemical smog causes serious health problems. Both ozone
and PAN act as powerful eye irritants. Ozone and nitric oxide irritate the nose and throat and their
high concentration causes headache, chest pain, dryness of the throat, cough and difficulty in
breathing. Photochemical smog leads to cracking of rubber and extensive damage to plant life. It
also causes corrosion of metals, stones, building materials, rubber and paint.
• Hence option (d) is the correct answer.

Q 9.A
• Recent Context - Indian astronomers collaborated with 2020 Physics Nobel Laureate Prof. Andrea Ghez
on Thirty Meter Telescope Project.
• About Thirty Metre Telescope (TMT):
o It is a proposed astronomical observatory with an extremely large telescope (ELT). The proposed site
for its installation is Maunakea in Hawaii. Hence statement 2 is not correct.
o The Thirty-meter telescope (TMT) project is an international partnership between CalTech,
Universities of California, Canada, Japan, China, and India. In India, it is being coordinated through
the Department of Science and Technology (DST) and Department of Atomic Energy (DAE)
o “Thirty Metre” refers to the 30-meter diameter of the mirror. This, along with 492 segments of glass
pieced together, will make TMT three times as wide as the world’s largest existing visible-light
telescope. The larger the mirror, the more light a telescope can collect, which means, in turn, that it
can “see” farther, fainter objects. It would be more than 200 times more sensitive than current
telescopes and would be able to resolve objects 12 times better than the Hubble Space
Telescope. Hence statement 1 is correct.
• Objectives
o When operational, it will help uncover hitherto unknown and even unexpected details of our vast
Universe.
o One of its key uses will be the study of exoplanets and whether their atmospheres contain water vapor
or methane — the signatures of possible life.
o The study of black holes is another objective. While these have been observed in detail within the
Milky Way, the next galaxy is 100 times farther away and the Thirty Metre Telescope will help bring
them closer.

Q 10.D
• The Animal Welfare Board of India (AWBI), the first of its kind a statutory body to be established by
any Government in the world, was set up in 1962, in accordance with Section 4 of the Prevention of
Cruelty to Animals Acts 1960, with headquarters at Chennai. Recently it headquarters was shifted at
Ballabhgarh in Haryana state.
• Shrimati Rukmini Devi Arundale pioneered the setting up of the Board. She guided the activities of the
Board for nearly twenty years till her demise in 1986.
• It is an advisory body advising the Government of India's Ministry of Fisheries, Animal Husbandry
and Dairying (Department of Animal Husbandry and Dairying). Its basic mandate is to advise the
government on animal welfare issues, and create awareness in animal welfare and to implement the
regular schemes of AWBI and also the central sector schemes for the welfare of animals. Hence the
correct answer is option (d).
4 www.visionias.in ©Vision IAS
Q 11.C
• Recent Context: The Information Management and Analysis Centre (IMAC) of the Indian Navy is set to
become a National Maritime Domain Awareness (NDMA) center. The proposed NDMA center will be a
multi-agency center and provide information to various stakeholders, from the fisheries department to
local policing authorities on development or movements across the coast. This will make sure that any
risk, especially transnational can be prevented.
• IMAC was set up in 2014 to provide coastal security and to avert tragic incidents like the 26/11 terror
attack on Mumbai.
o It the nodal center of the National Command Control Communications and Intelligence
Network (NC3I Network), and is a joint initiative of the Indian Navy, Coast Guard, and Bharat
Electronics Ltd.
• IMAC tracks only non-military or commercial ships, known as white shipping. Military ships, or grey
hull ships, are tracked by the Directorate of Naval Operations. Hence statement 2 is not correct.
• The IMAC is the center where data from various sensors and databases is aggregated, correlated,
and then disseminated to various stations for enhanced awareness. Hence statement 1 is correct.
• IMAC functions under the National Security Adviser (NSA). Hence statement 3 is correct.
• The NC3I network links 51 Naval and Coast Guard stations, located along the coast and on island
territories.
o The network provides these stations coastal surveillance information obtained from various sensors
such as the coastal radar chain of the Indian Coast Guard and automatic tracking systems as well as
electro-optical cameras.
o The network rides on dedicated terrestrial data circuits, as well as, satellite communication, which
helps the stations in remote locations to be networked.
o The entire NC3I Network has been integrated by Bharat Electronics Limited, Bangalore.
• The NC3I network and IMAC are also linked with the prestigious National Maritime Domain
Awareness (NMDA) project. In the NMDA project, the NC3I network will function as the
communication backbone and the IMAC will continue to be the nodal center but will be rechristened as
the NMDA Centre.
• Related Information:
o Automatic Identification Systems (AIS) transponders are fitted on merchant ships, air and traffic
management system, and global shipping databases.
o Following the 26/11 terror attack, fishing vessels that are more than 20m long were directed to have
AIS transponders installed.
o AIS is mainly intended to enhance the safety of life at sea (SOLAS) through safe navigation in the
marine environment. SOLAS regulations require that AIS system onboard exchange data ship-
to-ship and with shore-based facilities.
o The use of AIS is to help identify and track ships and SAR (Search and Rescue) operations and reduce
the mandatory verbal information exchange. With a digital display, shipborne AIS enables the
provision of automatic information by calculating the Closest Point of Approach (CPA) and Time to
Closest Point of Approach (TCPA).
o The International Maritime Organisation (IMO) publishes the guidelines for onboard
operational use of the shipborne Automatic Identification Systems (AIS).
o The Indian Ocean Region (IOR) is the commercial highway for large traffic and critical for the
prosperity of many nations.
o A regional repository of maritime data, along with IMAC is located in Delhi NCR and the IFC-IOR is
in lines with the principles listed under Security and Growth of All in the Region (SAGAR). It
presently has linkages with 21 partner countries and 22 multi-national agencies across the globe.

Q 12.C
• The Green Energy Corridor Project aims at synchronizing electricity produced from renewable
sources, such as solar and wind, with conventional power stations in the grid. For evacuation of
large-scale renewable energy, Intra State Transmission System (InSTS) project was sanctioned by the
Ministry in 2015-16.
• It is being implemented by eight renewable-rich states of Tamil Nadu, Rajasthan, Karnataka, Andhra
Pradesh, Maharashtra, Gujarat, Himachal Pradesh, and Madhya Pradesh. The project is being
implemented in these states by the respective State Transmission Utilities (STUs).

5 www.visionias.in ©Vision IAS


• The purpose is to evacuate approx. 20,000 MW of large scale renewable power and improvement of the
grid in the implementing States.
• The funding mechanism consists of 40% from the Government of India Grant (total Rs. 4056.67 crores),
20% from State Equity and 40% from the loan from KfW, Germany (500 million EUR).

Q 13.D
• Copper slag, which is produced during pyrometallurgical production of copper from copper ores contains
materials like iron, alumina, calcium oxide, silica etc. For every tonne of metal production about 2.2 ton
of slag is generated. Dumping or disposal of such huge quantities of slag causes environmental and space
problems.
• The favourable physio-mechanical characteristics of copper slag can be utilised to make the
products like cement, fill, ballast, abrasive, aggregate, roofing granules, glass, tiles etc. apart from
recovering the valuable metals by various extractive metallurgical routes. When used in hydraulically
bound applications, it increases workability and compressive and flexural strengths and decreases drying
shrinkage of concrete. It may also improve the abrasion resistance of concrete.
• Various processes such as pyro, hydro and a combination of pyro-hydrometallurgical methods are applied
for metal recovery and preparation of value-added products from copper slag. Hence the correct answer
is option (d).

Q 14.D
• Fly Ash is a fine powder, which is the by-product of burning coal in thermal power plants. It includes
substantial amounts of oxides of silica, aluminium and calcium. Elements like Arsenic, Boron, Chromium,
lead etc. are also found in trace concentrations.
• Owing to large-scale dependence on thermal power generation and high ash content in Indian coal, a large
quantity of ash is generated in the country (nearly 200 million tons). It not only requires a large area of
precious land for its disposal but is also one of the sources of air and water pollution.
• Advantages with fly ash utilisation:
o In agriculture: It improves water holding capacity, works as a soil conditioner and contains
micronutrients like phosphorus, potassium and calcium thus increasing the crop yield.
o In Construction works- Fly ash is a proven resource material for many applications of construction
industries and currently is being utilized in the manufacturing of portland cement, bricks/blocks/tiles
manufacturing, road embankment construction and low-lying area development, etc.
▪ Concrete made with fly ash is stronger and more durable than traditional concrete made
with Portland cement.
▪ Fly ash is a lightweight material and therefore it undergoes lesser settlement and hence can be
used for embankment construction over a weak substrate such as alluvial clay or silt where
excessive weight could cause failure.
o In the manufacturing of Absorbents that are suitable for purification of waste gases, drinking
water purification, wastewater treatment etc.
o In preventing contamination of Water Resources- by preventing contamination of surface water
through erosion, runoff, airborne particles landing on the water surface etc.
• Hence, option (d) is correct.

Q 15.B
• The National Green Tribunal was established in 2010 under the National Green Tribunal Act, 2010 for
effective and expeditious disposal of cases relating to environmental protection and conservation of
forests and other natural resources.
• The Tribunal is competent to hear matters relating to laws listed in Schedule I of the NGT Act. These are:
o Water (Prevention and Control of Pollution) Act, 1974.
o The Water (Prevention and Control of Pollution) Cess Act, 1977
o The Forest (Conservation) Act, 1980.
o The Air (Prevention and Control of Pollution) Act, 1981.
o The Environment (Protection) Act, 1986.
o The Public Liability Insurance Act, 1991.
o The Biological Diversity Act, 2002.

6 www.visionias.in ©Vision IAS


• Some significant laws are not included:
o Wildlife (Protection) Act, 1972 which prevents any sort of destruction inside sanctuaries without
permission.
o Scheduled Tribes (Recognition of Forest Rights) Act, 2005 which gives rights to forest-dwelling
Scheduled Tribes to protect and conserve forest resources.
o Hence option (b) is the correct answer.

Q 16.D
• International Convention for the Prevention of Pollution from Ships (MARPOL Convention) is one of the
most significant international marine environmental conventions.
• The International Convention for the Prevention of Pollution from Ships (MARPOL) is the main
international convention covering prevention of pollution of the marine environment by ships from
operational or accidental causes.
• The Convention includes regulations aimed at preventing and minimizing pollution from ships -
both accidental pollution and that from routine operations - and currently includes six technical
Annexes. Hence, statement 1 is correct.
• The MARPOL Convention was adopted on 2 November 1973 at IMO. The Protocol of 1978 was
adopted in response to a spate of tanker accidents in 1976-1977.
• The current convention is a combination of the 1973 Convention and the 1978 Protocol, which entered
into force on 2 October 1983. The Convention includes regulations aimed at preventing and minimizing
pollution from ships - and currently includes six technical Annexes.
• MARPOL has been updated by amendments through the years. In 1997, a Protocol was adopted to
amend the Convention and a new Annex VI (Prevention of Air Pollution from Ships) was added
which entered into force on 19 May 2005. It sets limits on sulphur oxide and nitrogen oxide emissions
from ship exhausts and prohibits deliberate emissions of ozone-depleting substances; designated emission
control areas set more stringent standards for SOx, NOx and particulate matter. A chapter adopted in 2011
covers mandatory technical and operational energy efficiency measures aimed at reducing greenhouse gas
emissions from ships. Hence statement 2 is correct.
• India has signed the International Convention for the Prevention of Pollution from Ships
(MARPOL). Hence, statement 3 is not correct.

Q 17.C
• Recent Context - The blooms of dinoflagellates (Noctiluca Scintillans), commonly known as “sea
sparkle” are being witnessed along the coasts of Maharashtra and Karnataka.
• Noctiluca Scintillans
o It is a free-living, marine-dwelling species of dinoflagellate that exhibits bioluminescence when
disturbed (popularly known as mareel). Hence option (c) is the correct answer.
o N. Scintillans grazes on other micro-organisms such as larvae, fish eggs, and diatoms and thus affect
the marine food system. Also, the unicellular phytoplankton that lives inside it can photosynthesize,
turning sunlight into energy. They help their host cell survive even when food was scarce. Thus, N.
Scintillans acts as both a plant and an animal. Hence statement 1 is correct.
o The toxic blooms of N. Scintillans are being linked to massive fish and marine invertebrate kills.
o Though the species does not produce a toxin, it was found to accumulate toxic levels of ammonia,
which is then excreted into the surrounding waters, possibly acting as the killing agent in blooms.
• Bioluminescence:
o It is the production and emission of light by a living organism.
o The light emitted by a bioluminescent organism is produced by energy released from chemical
reactions occurring inside (or ejected by) the organism.

Q 18.B
• Radiation refers to the energy emitted from a body or source. Radiation is classified as being either non-
ionizing or ionizing.
• Non-ionizing radiation is longer wavelength/lower frequency lower energy. While ionizing radiation is
short-wavelength/high frequency higher energy.
• Non-Ionizing radiation forms: ELF (extremely low frequency or High wavelength), Radio
frequencies, Microwave frequencies, Lasers, Infrared, Visible, Spectrum, Ultraviolet rays. The injury if
at all caused by ionizing radiation, it is limited to burns.
• Ionizing radiation is emitted from radioactive atomic structures as high energy electromagnetic waves
(gamma and x-rays) or as actual particles (alpha, beta particles)
7 www.visionias.in ©Vision IAS
o Gamma Rays - can penetrate easily to human skin and damage cells on its way through, reaching far,
and can only be blocked by a very thick, strong, massive piece of concrete.
o X-rays - can pass through most objects including the human body.
o Alpha Particles - They can be blocked by a piece of paper and human skin.
o Beta Particles - Beta particles can penetrate through skin, while can be blocked by some pieces of
glass and metal.
• Ionizing radiations have high penetration power and cause breakage of macromolecules. Short-range
effects include burns, impaired metabolism, dead tissues and death of the organisms. Long-range effects
are mutations increased the incidence of tumors and cancer, shortening of life-span and developmental
changes. The mutated gene can persist in living organisms and may affect their progeny.

Q 19.D
• To address the issue of ensuring environmentally sound management of hazardous waste for the safety of
health and environment during handling of such waste Hazardous Waste (Management, Handling &
Transboundary Movement) Rules, 2008 were notified under Environment (Protection) Act, 1986.
• The Rules lay down the procedure towards this process by providing provisions for authorization of
hazardous waste generating and units using hazardous waste. It also provides for the establishment of a
Treatment Storage and Disposal Facility (TSDF) for the disposal of hazardous wastes.
• The rules have an important provision on the regulation of import/export of hazardous waste in pursuance
of our obligation under the Basel Convention on Control of Transboundary Movement of Hazardous
waste and its disposal. India is a party to the Convention.
• The major salient features of Hazardous and Other Wastes (Management and Transboundary
Movement) Rules, 2016 include the following:-
o The ambit of the Rules has been expanded by including ‘Other-Waste’.;
o Authorization and registration requirement is replaced with one permission i.e., authorization under
the rules for all stakeholders handling the hazardous and other waste; waste management hierarchy in
the sequence of priority as prevention, minimization, reuse, recycling, recovery, co-processing; and
safe disposal being incorporated;
o The process of import/export of waste under the rules has been streamlined by simplifying the
procedure and by revising the list of waste regulated for import/export;
o A separate schedule has been introduced which comprises waste such as metal scrap, paper waste and
various categories of electrical and electronic equipments for re-use purpose exempted from the need
of obtaining Ministry’s permission;
o The list of waste prohibited for import has been revised by inclusion of following items: waste
edible fats and oils of animals or vegetable origin, household waste, critical care medical
equipment, solid plastic wastes, other chemical wastes especially in solvent form.
• Hence option (d) is the correct answer.

Q 20.B
• Methanol Economy program initiated by NITI Aayog is aimed at reducing our oil import bill; reducing
greenhouse gas (GHG) emissions & converting Indian coal reserves and municipal solid waste into
methanol leading to independence from import while creating new jobs by setting up Methanol
Production Plants.
• “Methanol as a low carbon, hydrogen carrier fuel” produced from high ash coal, agricultural
residue, CO2 from thermal power plants and natural gas is the best pathway for meeting India’s
commitment to COP 21. Hence the correct answer is option (b).
• Methanol, although slightly lower in energy content than petrol and diesel, can replace both petrol and
diesel in the transport sector (road, rail and marine), the energy sector (comprising of DG sets, boilers,
process heating modules, tractors and commercial vehicles) and retail cooking replacing LPG (partially),
Kerosene and wood charcoal.
• Blending of 15% methanol in gasoline can result in at least a 15% reduction in the import of gasoline/
crude. In addition, it would bring down GHG emissions by 20% in terms of particulate matter, NOx, and
SOx thereby improving urban air quality.

Q 21.C
• Vitrification is an engineering approach to soil amendment. During vitrification, contaminated soil mass
is treated through the processes involving melting and refreezing to create a glass-like solid that entraps
inorganic contaminants and thereby isolates these from the environment. Hence option (c) is the correct
answer.
8 www.visionias.in ©Vision IAS
• It can be applied both in-situ and elsewhere above ground in a treatment unit (ex-situ). The high
temperature applied during the vitrification process also destroys organic contaminants in soil.
Vitrification is thus applicable for treating soil contaminated with both toxic organic chemicals and trace
elements.
• The process involves the introduction of electrodes into the wet soil that is capable of carrying current.
Heat generated during current flow raises the temperature of surrounding soil mass which is sufficient
to liquefy siliceous soil particles. After melting is over, the contaminated mass is solidified upon cooling.
Toxic gases can also be produced during vitrification of soil contaminated with certain contaminants.
Full-scale vitrification technologies exist for soils contaminated with arsenic, lead and chromium. Soil
mixed with various toxic wastes can also be treated in this manner.

Q 22.B
• Biomagnification is the process by which toxic chemicals build up within predators. This typically occurs
across an entire food chain and affects all of the organisms but animals higher up in the chain are more
impacted. When predatory animals consume their prey they also consume all of the toxic chemicals within
said prey.
• When these toxins aren't easily excreted they build up in the animal's system through bioaccumulation.
Therefore, when the food chain progresses, concentrations increase or magnify. Biomagnification can be
considered the result of bioaccumulation. Some of the substances that cause Biomagnification include:
o Mercury is a persistent substance, it can build up, or bioaccumulate, in living organisms, inflicting
increasing levels of harm on higher-order species such as predatory fish and fish-eating birds and
mammals through a process known as "biomagnification". Although the long-term effects of mercury
on whole ecosystems are unclear, the survival of some affected populations and overall biodiversity is
at risk
o The concentration-effect occurs because DDT is metabolized and excreted much more slowly than the
nutrients that are passed from one trophic level to the next. So DDT accumulates in the bodies
(especially in fat). Thus most of the DDT remains at that trophic level. This is why the hazard of DDT
to nontarget animals is particularly acute for those species living at the top of food chains.
o Polychlorinated biphenyls (PCBs) tend to build up in living organisms both by uptake from the
environment over time (bioaccumulation) and along the food chain (biomagnification). PCBs remain
stored in fatty tissues much more than in muscles or other body parts.
• However, no data pertaining to the potential of sulfur dioxide to bioaccumulate or biomagnify in the
food chain were identified. Hence, option (b) is correct.

Q 23.A
• Statement 1 is correct: Environmental Impact Assessment is defined as an activity that has been
designed to identify, predict and interpret the impact of an action on human health, including the well
being of the ecosystem on which the survival of human beings depends. Environment Impact Assessment
in India is statutorily backed by the Environment Protection Act in 1986, which contains various
provisions on EIA methodology and process.
• All projects and activities are broadly categorized into two categories - Category A and category B, based
on the spatial extent of potential impacts and potential impacts on human health and natural and man-
made resources.
o All projects or activities included as Category ‘A’ in the Schedule require prior environmental
clearance from the Central Government in the Ministry of Environment and Forests (MoEF) on the
recommendations of an Expert Appraisal Committee (EAC) to be constituted by the Central
Government.
o All projects or activities included as Category ‘B’ in the Schedule require prior environmental
clearance from the State/Union territory Environment Impact Assessment Authority (SEIAA). Hence
statement 2 is not correct.
• The Public Hearing under EIA is arranged in a systematic, time-bound and transparent manner ensuring
widest possible public participation at the project site(s) or in its close proximity District -wise, by the
concerned State Pollution Control Board (SPCB) or the Union Territory Pollution Control Committee
(UTPCC). There is no quorum required for attendance for starting the proceedings.
• Statement 3 is not correct: The Public Consultation shall ordinarily have two components comprising
of:-(a) a public hearing at the site or in its close proximity- district wise, to be carried out in the manner
prescribed in Appendix IV, for ascertaining concerns of local affected persons;(b) obtain responses in
writing from other concerned persons having a plausible stake in the environmental aspects of the project
or activity.
9 www.visionias.in ©Vision IAS
Q 24.A
• Recent Context - United Nations (UN) in collaboration with The Vaccine Confidence Project at the
University of London has launched ‘Team Halo initiative.
• About Team Halo Initiative: The United Nations has collaborated with The Vaccine Confidence Project
at the University of London’s School of Hygiene and Tropical Medicine to undertake the Team Halo
initiative.
o It aims to use social media to counter the misinformation around COVID-19 vaccines and build
confidence by busting myths and sharing information on the safety and effectiveness of vaccines. The
‘halo’ represents the ring of connected science that circles the globe. Hence option (a) is the correct
answer.
o Globally, the initiative is supported by over 100 scientists who are involved in the race of coronavirus
vaccine development across the world’s top institutes.
o In India, over 22 scientists have joined Team Halo from renowned institutes such as the Institute of
Medical Sciences and Sum Hospital; PGIMER, Nizams Institute of Medical Sciences, IIT Indore,
Christian Medical College, SRM Research Institute, and Deep Children's Hospital and Research
Centre.
o Team Halo India will also allow scientists to respond to questions from the public and directly counter
vaccine misinformation and rumours spreading in parts of the internet.

Q 25.A
• Ocean Thermal Energy Conversion uses the ocean thermal gradient between cooler deep and
warmer shallow or surface seawaters to run a heat engine and produce useful work, usually in the
form of electricity. OTEC can operate with a very high capacity factor and so can operate in base load
mode.
• Among ocean energy sources, OTEC is one of the continuously available renewable energy resources that
could contribute to base-load power supply. The resource potential for OTEC is considered to be much
larger than for other ocean energy forms.[2] Up to 88,000 TWh/yr of power could be generated from
OTEC without affecting the ocean's thermal structure.

Q 26.C
• Recent Context: PM of India has recently chaired the PRAGATI meeting. It is held once every month
on Fourth Wednesday at 3.30 PM and known as PRAGATI Day.
• PRAGATI (Pro-Active Governance And Timely Implementation) is also a robust system for bringing
e-transparency and e-accountability with real-time presence and exchange among the key stakeholders.
• It is a multi-purpose and multi-modal platform that is aimed at addressing the common man’s grievances
and simultaneously monitoring and reviewing important programs and projects of the Government of
India as well as projects flagged by State Governments. Hence statement 1 is correct.
• It was launched in 2015 and has been designed by the Prime Minister’s Office (PMO) team with the help
of the National Informatics Center (NIC).
• It is a three-tier system (PMO, Union Government Secretaries, and Chief Secretaries of the States).
• The three objectives of PRAGATI are:
o Grievance Redressal
o Program Implementation
o Project Monitoring
• The PRAGATI platform uniquely bundles the three latest technologies: Digital data management, video-
conferencing, and geospatial technology. Hence statement 2 is correct.

Q 27.A
• The Graded Response Action Plan (GRAP), is a set of curbs triggered in phases as the air
quality deteriorates, which is typical of the October-November period in the Delhi-NCR region. Hence
option (a) is the correct answer.
• Approved by the Supreme Court in 2016, the plan was formulated after several meetings that the
Environment Pollution (Prevention and Control) Authority held with state governments
representatives and experts. The result was a plan that institutionalised measures to be taken when air
quality deteriorates.
• GRAP works only as an emergency measure. The plan is incremental in nature and measures are
following according to air quality. If air quality reaches the severe+ stage, GRAP talks about shutting
down schools and implementing the odd-even road-space rationing scheme. The plan requires action
and coordination among 13 different agencies in Delhi, Uttar Pradesh, Haryana and Rajasthan (NCR
areas).

10 www.visionias.in ©Vision IAS


Q 28.C
• With an objective to conserve the Olive Ridley Turtles, the UNDP Sea Turtle Project was initiated
by Wildlife Institute of India, Dehradun as the Implementing Agency in November 1999.
• The following species of marine are being covered under this project - green, hawksbill, loggerhead,
leatherback, and olive ridley. Hence both statements 1 and 2 are not correct.
• One of the main aims of the GOI – UNDP national sea turtle project was to evaluate the status of and
threats to marine turtles along the entire coastline of India.
• The project is for 10 coastal states in India especially Odisha where it has contributed towards the
preparation of a map of breeding sites of Sea Turtles; identification of breeding places and habitats along
the coastline, and migratory routes taken by Sea Turtles. Hence statement 3 is correct.
• The project also helped in the development of guidelines to safeguard the turtle mortality rate and for
tourism in sea turtle areas. Amongst the major achievements of the project is the demonstration of the use
of Satellite Telemetry to locate the migratory route of sea turtles in the sea.

Q 29.D
• The Ramsar Convention, ratified by Government of India, defines wetlands as ‘areas of marsh, fen,
peatland or water, whether natural or artificial, permanent or temporary, with water that is static or
flowing, fresh, brackish or salt, including areas of marine water the depth of which, at low tides, does not
exceed six meters’. In addition, the Convention provides that ‘wetlands may include riparian and coastal
zones adjacent to the wetlands, and islands or bodies of marine water deeper than six meters at low tide
lying within the wetlands.’ Hence option (d) is the correct answer.

Q 30.A
• The common components of photochemical smog are ozone, nitric oxide, acrolein, formaldehyde
and peroxyacetyl nitrate (PAN). Hence statement (1) is correct.
• Photochemical smog occurs in warm, dry and sunny climate. The main components of the
photochemical smog result from the action of sunlight on unsaturated hydrocarbons and nitrogen oxides
produced by automobiles and factories. Hence statement (2) is not correct.
• Usually catalytic converters are used in the automobiles, which prevent the release of nitrogen oxide and
hydrocarbons to the atmosphere.
• Certain plants e.g., Pinus, Juniparus, Quercus, Pyrus and Vitis can metabolise nitrogen oxide and
therefore, their plantation could help in this matter. Hence statement (3) is correct.

Q 31.C
• Recent Context - The Food and Agriculture Organization of the United Nations formally launched the
Food Coalition.
• The Background: The novel coronavirus disease (COVID-19) pandemic has pulled food systems apart,
threatening food security and nutrition. At least 690 million people went hungry in 2019, according to the
State of Food Security and Nutrition in the World report, 2020. Now, the pandemic could tip over 130
million more people into chronic hunger by the end of 2020.
o Taking cognizance of the catastrophic food crisis and the urgency to tackle it, the Food and
Agriculture Organization (FAO) of the United Nations launched a Food Coalition on November 5,
2020. FAO Director-General virtually inaugurated it along with the Prime Minister of Italy, Giuseppe
Conte, and the Deputy Prime Minister of the Netherlands, Carola Schouten.
• About Food Coalition: It has been launched by the Food and Agriculture Organization of the
United Nations. Hence statement 1 is correct.
o It is a voluntary multi-stakeholder and multi-sectoral alliance set up to support innovative initiatives
to ensure global food access, increase the resilience of agri-food systems, and put them on a more
sustainable course.
o It is a network of networks for unified global action in response to COVID-19 with respect to
food access. Hence statement 2 is correct. The Food Coalition aims to work for unified global
action in response to COVID-19 and the risks to agri-food systems it poses.
o The alliance involves a devoted trust fund and a web-based hub allowing participants to access a
basket of project-focused information and data, as well as the funding and types of assistance needed
for many on-the-ground projects.
o It was proposed by Italy. Italy has supported the alliance with an initial contribution of $1.2 million.
11 www.visionias.in ©Vision IAS
• The Food Coalition would work towards:
o Mobilizing resources, expertise, and innovation
o Advocating for a joint and coordinated COVID-19 response
o Promoting dialogue and exchange of knowledge and expertise among countries
o Working towards solution-oriented plans and programs
o Expanding international cooperation and partnership for a long term impact

Q 32.C
• Microbeads are plastic pieces or fibre measuring less than 5 mm. They are used in personal care
products and are made of polyethylene (PE), but can also be made of polypropylene (PP), polyethene
terephthalate (PET), polymethyl methacrylate (PMMA) and nylon. Hence, statement 1 is correct.
• These tiny pieces of plastic don’t dissolve in water, which is why they work so well at cleaning the oil
and dirt out of pores. However, because they are so small (less than 1 millimetre), the microbeads are not
filtered out by wastewater treatment plants. Hence, statement 2 is not correct.
• They are widely used in cosmetics as exfoliating agents and in personal care products such as toothpaste,
as well as in biomedical research. Research studies have also shown its presence in table salt.
• They are so small that a person can barely feel them. Their roundness and particle size creates a
ballbearing effect in creams and lotions resulting in a silky texture and spreadability.
• Once in the water, microbeads can have a damaging effect on marine life, the environment and
human health. This is due to their composition, ability to adsorb toxins and potential to transfer up
the marine food chain. These tiny plastics persist in the environment as they are almost impossible to
remove. Hence, statement 3 is correct.

Q 33.A
• The Seventh Conference of Parties (COP-7) to the UNFCCC decided that Parties participating in Clean
Development Mechanism (CDM) should designate a National Authority for the CDM and as per the
CDM project cycle, a project proposal should include written approval of voluntary participation from the
Designated National Authority of each country and confirmation that the project activity assists the host
country in achieving sustainable development.
• Accordingly, the Central Government constituted the National Clean Development Mechanism (CDM)
Authority for the purpose of protecting and improving the quality of the environment in terms of the
Kyoto Protocol.
• Secretary (Environment and Forests) is the chairperson of the National Clean Development Mechanism
Authority. Hence option (a) is the correct answer.
• The National Clean Development Mechanism (CDM) Authority receives projects for evaluation and
approval as per the guidelines and general criteria laid down in the relevant rules and modalities
pertaining to CDM.
• The National Clean Development Mechanism (CDM) Authority can recommend certain additional
requirements to ensure that the project proposals meet the national sustainable development priorities and
comply with the legal framework so as to ensure that the projects are compatible with the local priorities
and stakeholders have been duly consulted.
• The Authority also carries out the financial review of project proposals to ensure that the project
proposals do not involve diversion of official development assistance in accordance with modalities
and procedures for Clean Development Mechanism.
• The National Clean Development Mechanism (CDM) Authority has the powers:
o To invite officials and experts from Government, financial institutions, consultancy organizations,
non-governmental organizations, civil society, the legal profession, industry and commerce, as it may
deem necessary for technical and professional inputs and may co-opt other members depending upon
need.
o To interact with concerned authorities, institutions, individual stakeholders for matters relating to
CDM.
o To take up any environmental issues pertaining to CDM or Sustainable Development projects as may
be referred to it by the Central Government.

Q 34.D
• Energy Efficiency Services Limited (EESL), a joint venture of PSUs under the Ministry of Power and
Department of New & Renewable Energy (DNRE), Goa, have signed a memorandum of understanding
today to discuss rolling out of India’s first Convergence Project in the State.

12 www.visionias.in ©Vision IAS


• Under the MoU, EESL and DNRE will carry-out the feasibility studies and subsequent implementation of
decentralized solar energy projects. EESL shall implement the solar energy projects, the
establishment of 100 MW of decentralized ground-mounted Solar Power projects on government
lands to be used for agricultural pumping, replace approximately 6,300 agricultural pumps with
BEE star rated energy efficient pumps and distribute approximately 16 Lakh LED bulbs for rural
domestic households.
• This model is expected to be adopted by other states as this will reduces losses in terms of expenditure on
water for farm sector runs into tens of thousands of crore rupees in several States. That preempts state
spending on health, education, and other important sectors.
• The projects will accelerate the usage of renewable energy sources, especially for agricultural and rural
power consumption in the State. The projects will also contribute in the reduction of peak energy demand
through the deployment of energy-efficient pumping and lighting thus contributing to overall
sustainability. Hence the correct answer is option (d)

Q 35.A
• Recent Context – Recently, the European Space Agency formally adopted the Ariel Space Mission. The
mission has moved from study to implementation phase, following which an industrial contractor will be
selected to build the spacecraft.
• About the ARIEL mission:
o Ariel stands for the Atmospheric Remote-sensing Infrared Exoplanet Large-survey.
o It is a space exploration mission that will study the composition, nature, and evolution of about 1000
extrasolar planets, simultaneously in visible and infrared wavelengths. Hence statement 1 is correct.
o It is the first mission dedicated to measuring the chemical composition and thermal structures of
transiting exoplanets which lie far beyond the boundaries of the Solar System.
o It will be launched in 2029 and the duration of the mission will be 4 years.
o It will operate from an orbit around the second Sun-Earth Lagrange point, L2, 1.5 million kilometers
directly ‘behind’ Earth as viewed from the Sun, on an initial four-year mission.
o The mission’s payload module, which includes one meter-class cryogenic telescope and associated
science instruments, is provided by the Ariel Mission Consortium. The consortium comprises more
than 50 institutes from 17 European countries including UK, France, Italy, Germany, the
Netherlands, Poland, Spain, Belgium, Austria, Denmark, Ireland, Portugal. NASA also
contributes to the payload. Hence statement 2 is not correct.
• Exoplanets
o These are the planets that lie outside of the Solar System and orbit around stars other than the Sun.
They are also called Extrasolar planets.
o They are not easy to detect due to their less brightness than the stars they orbit and hence it is difficult
to see them directly using telescopes.
o As per NASA, only a handful of exoplanets have been found using telescopes and the rest have been
detected using indirect methods. One of these methods involves tracking the dimming of a star that
happens when a planet passes in front of it. NASA’s Kepler Space Telescope uses this method to spot
thousands of planets.
o Other methods to track exoplanets include gravitational lensing and the ‘wobbling method which is
based on the idea that an orbiting planet will cause its parent star to orbit slightly off center.
o Scientists study the Exoplanets to find the possibility of life beyond Earth. It helps in understanding
the functioning of stars and planets in the solar system. It helps in understanding the evolution of
space and life.

Q 36.C
• The Cartagena Protocol on Biosafety to the Convention on Biological Diversity is an international
treaty governing the movements of living modified organisms (LMOs) resulting from modern
biotechnology from one country to another.
• It is a supplement to the Convention on Biological Diversity. It aims to ensure the safe handling, transport
and use of organisms that have been modified using modern biotechnology.
• The Protocol aims to guard against adverse effects on biological diversity, also taking also into account
risks to human health. The Protocol, which entered into force in 2003.
• It establishes an advance informed agreement (AIA) procedure for ensuring that countries are provided
with the information necessary to make informed decisions before agreeing to the import of such
organisms into their territory.

13 www.visionias.in ©Vision IAS


• The Protocol also establishes a Biosafety Clearing-House to facilitate the exchange of information
on living modified organisms and to assist countries in the implementation of the Protocol.
• UN Environment works in partnership with the Global Environment Facility and the Convention on
Biological Diversity to assist the Parties to the Cartagena Protocol to meet their commitments under the
agreement.
• Hence both the statements are correct.

Q 37.D
• The Global Environment Facility was established on the eve of the 1992 Rio Earth Summit to help tackle
our planet‘s most pressing environmental problems. Since then, the GEF has provided over $17.9 billion
in grants and mobilized an additional $93.2 billion in co-financing for more than 4500 projects in 170
countries.
• Today, the GEF is an international partnership of 183 countries, international institutions, civil society
organizations and the private sector that addresses global environmental issues. The GEF provides
funding to assist developing countries in meeting the objectives of international environmental
conventions.
• The GEF serves as "financial mechanism" to five conventions, which are UN Convention on Biological
Diversity (CBD), United Nations Framework Convention on Climate Change (UNFCCC),
Stockholm Convention on Persistent Organic Pollutants (POPs), UN Convention to Combat
Desertification (UNCCD), and Minamata Convention on Mercury.
• The GEF, although not linked formally to the Montreal Protocol, supports the implementation of the
Protocol in Countries with Economies in Transition.
• Hence option (d) is the correct answer.

Q 38.C
• Acid rain refers to a mixture of deposited material, both wet and dry, coming from the atmosphere
containing more than normal amounts of nitric and sulfuric acids. Simply put, it means rain that is
acidic in nature due to the presence of certain pollutants in the air due to cars and industrial
processes.
• Causes of Acid Rain
o Both natural and man-made sources are known to play a role in the formation of acid rain. But, it is
mainly caused by the combustion of fossil fuels which results in emissions of sulfur dioxide (SO2)
and nitrogen oxides (NOx).
o Natural Sources:
▪ The major natural causal agent for acid rain is volcanic emissions.
▪ Decaying vegetation, wildfires and biological processes within the environment also generate acid
rain forming gases.
▪ Lighting strikes also naturally produce nitric oxides that react with water molecules via electrical
activity to produce nitric acid, thereby forming acid rain.
o Man-made Sources:
▪ Human activities leading to chemical gas emissions such as sulfur and nitrogen are the primary
contributors to acid rain. The activities include air pollution sources emitting sulfur and nitrogen
gases like factories, power generation facilities, and automobiles.
• Effects of Acid Rain: Acid rain has significant effects on the world environment and public health.
o Effect on Aquatic Environment: Acid rain either falls directly on aquatic bodies or gets run off the
forests, roads and fields to flow into streams, rivers and lakes. Over a period of time, acids get
accumulated in the water and lower the overall pH of the water body. The aquatic plants and animals
need a particular pH level of about 4.8.
o Effect on Forests: It makes trees vulnerable to disease, extreme weather, and insects by destroying
their leaves, damaging the bark and arresting their growth.
o Effect on Soil: Acid rain highly impacts on soil chemistry and biology. It means soil microbes and
biological activity, as well as soil chemical compositions such as soil pH, are damaged or reversed
due to the effects of acid rain.
o Effect on Architecture and Buildings: Acid rain on buildings, especially those constructed with
limestone, reacts with the minerals and corrode them away.
o Effect on Public Health: When in the atmosphere, sulfur dioxide and nitrogen oxide gases and their
particulate matter derivatives like sulfates and nitrates, degrades visibility and can cause accidents,
leading to injuries and deaths. Human health is not directly affected by acid rain because acid
rainwater is too dilute to cause serious health problems.
14 www.visionias.in ©Vision IAS
• Solutions to Acid Rain
o Cleaning up Exhaust Pipes and Smokestacks, Washing coal, use of coal comprised of low sulfur,
and use of devices known as “scrubbers” can provide technical solutions to SO2 emissions.
o Restoring Damaged Environments: The use of limestone or lime, a process called liming, is a
practice that people can do to repair the damage caused by acid rain to lakes, rivers and
brooks. Adding lime into acidic surface waters balances the acidity.
o Alternative Energy Sources: Besides fossil fuels, there is a wide range of alternative energy sources
that can generate electrical power. These include wind energy, geothermal energy, solar energy,
hydropower, and nuclear power. Fuel cells, natural gas, and batteries can also substitute the use of
fossil fuel as cleaner energy sources. Hence the correct option is (c)

Q 39.D
• Hazardous waste means any waste, which by reason of characteristics, such as physical, chemical,
biological, reactive, toxic, flammable, explosive or corrosive, causes danger to health, or environment. It
comprises the waste generated during the manufacturing processes of the commercial products such as
industries involved in petroleum refining, production of pharmaceuticals, petroleum, paint,
aluminum, electronic products etc.
• Unscientific disposal of hazardous and other waste through burning or incineration leads to emission of
toxic fumes comprising of Dioxins & Furans, Mercury, heavy metals, causing air, water pollution and
associated health-related problems.
• Hazardous and Other Wastes (Management & Transboundary Movement) Rules, 2016 were
promulgated by the Central Government to strengthen the implementation of environmentally sound
management of hazardous waste in the country.
• International Conventions on hazardous waste management are
• Basel Convention:
o The Basel Convention on the Control of Transboundary Movements of Hazardous Wastes and their
Disposal first came into force in 1992. The Convention puts an onus on exporting countries to ensure
that hazardous wastes are managed in an environmentally sound manner in the country of import.
• Stockholm Convention:
o The Stockholm Convention on Persistent Organic Pollutants aims to eliminate or restrict the
production and use of all intentionally produced POPs found in industrial chemicals and pesticides.
India signed the Convention in 2002 and ratified it in 2006. Recently the environment ministry has
notified new Regulation of Persistent Organic Pollutants (POP) Rules, 2018 which ban the
manufacture, trade, use, import and export of the seven toxic chemicals listed under the Stockholm
Convention.
• Rotterdam Convention:
o Rotterdam Convention on the Prior Informed Consent Procedure for Certain Hazardous
Chemicals and Pesticides in International Trade is a multilateral treaty to promote shared
responsibilities in relation to importation of hazardous chemicals. The convention promotes open
exchange of information and calls on exporters of hazardous chemicals to use proper labeling,
include directions on safe handling, and inform purchasers of any known restrictions or bans.
• India is a signatory to all the three conventions. Hence option (d) is the correct answer.

Q 40.A
• Persistent organic pollutants (POPs) are a class of highly hazardous chemical pollutants that are
recognized as a serious, global threat to human health and to ecosystems. POPs are substances that
specifically: remain intact for exceptionally long periods of time (many years).
• Some POPs are pesticides; some are industrial chemicals; and some are unintentionally produced
byproducts that are formed during certain combustion and chemical industry processes. Some examples of
POPs are DDT, lindane, PCBs and dioxins.
• POPs in the environment pollute the everyday food supply, especially fish, meat, butter and cheese. When
people eat POPs-contaminated foods, the POPs accumulate in their fatty tissue. They have the potential to
cause cancers, tumours, neurological disorders, reproductive disaster etc.
• Stockholm Convention on Persistent Organic Pollutants was signed in 2001 to eliminate or restrict the
production and use of persistent organic pollutants (POPs). Initiall 12 POPs Initially, twelve POPs have
been recognized as causing adverse effects on humans and the ecosystem which include

15 www.visionias.in ©Vision IAS


o pesticides like aldrin, chlordane, DDT, dieldrin, endrin, heptachlor, hexachlorobenzene, mirex,
toxaphene and
o Industrial chemicals like hexachlorobenzene, polychlorinated biphenyls (PCBs); and
o By-products like hexachlorobenzene etc.
• India had ratified the Stockholm Convention in 2006. Considering its commitment to the
implementation of the Convention, recently the Central Government had ratified prohibition of
manufacture, trade, use, import and export of the following seven POPs under the 'Regulation of
Persistent Organic Pollutants Rules (under the provisions of Environment (Protection) Act, 1986) -
Chlordecone, Hexabromobiphenyl, Hexabromodiphenyl ether and Heptabromodiphenylether
(Commercial octa-BDE), Tetrabromodiphenyl ether and Pentabromodiphenyl ether,
Pentachlorobenzene, Hexabromocyclododecane, and Hexachlorobutadiene. Hence option (a) is the
correct answer.

Q 41.D
• Incineration is a waste treatment process that involves the combustion of organic substances
contained in waste materials. Incineration and other high-temperature waste treatment systems are
described as "thermal treatment". Hence the correct option is (d)
• Incineration of waste materials converts the waste into ash, flue gas and heat. The ash is mostly formed by
the inorganic constituents of the waste and may take the form of solid lumps or particulates carried by the
flue gas. The flue gases must be cleaned of gaseous and particulate pollutants before they are dispersed
into the atmosphere. In some cases, the heat that is generated by incineration can be used to generate
electric power.
• As of July 2017, thermal-based waste-to-energy (WTE) plants in India have the capacity to process
5,300 tonnes of garbage and produce 63.5 MW per day.
• According to a 2015-16 report by the Ministry of New and Renewable Energy (MNRE), this capacity
can be enhanced to 1,075 MW by 2031 and to 2,780 MW by 2050.
• The new policy seeks to increase the energy generated from solid waste to 330 MW per day in the next
five months, and over 400 percent increase from the current installed capacity, and to 511 MW by 2018-
19. To this end, five WTE plants are already under construction in the country and tenders have been
floated to build another 47.

Q 42.A
• The Standards & Labeling Programme is one of the major thrust areas of the Bureau of Energy
Efficiency (BEE).
• A key objective of this scheme is to provide the consumer an informed choice about the energy-saving
and thereby the cost-saving potential of the relevant marketed product. The scheme targets display of
energy performance labels on high energy end-use equipment & appliances and lays down minimum
energy performance standards.
• Presently, the S&L program covers star rating for 26 appliances/equipment which is grouped into two
categories: mandatory and voluntary
• The following products have been notified under mandatory labeling.
o Frost Free (No-Frost) Refrigerator
o Tubular Fluorescent Lamps
o Room Air Conditioners
o Distribution Transformers
o Room Air Conditioners (Cassette, Floor Standing Tower, Ceiling, Corner AC)
o Direct Cool Refrigerator
o Electric Geysers
o Color TV
o Room Air Conditioners (Inverter type)
o LED lamps
• Products under voluntary labeling
o Induction Motors
o Pump sets
o Ceiling fans
o Liquefied Petroleum Gas (LPG) Stoves
o Washing machine
o Computer (Notebook/Laptops)
o Ballast (Electronic/Magnetic)
16 www.visionias.in ©Vision IAS
o Office equipment's (Printer, Copier, Scanner, MFD's)
o Diesel Engine Driven Monoset Pumps for Agricultural Purposes
o Solid State Inventor
o Diesel Generator
o Chillers
o Microwave Ovens
o Deep Freezers
o Light Commercial Air Conditioners (LCAC)
o Solar Water Heater.
• Hence the correct answer is option (a)

Q 43.B
• Statement 1 is not correct: Hydropower projects are classified as large and small hydro projects based
on their sizes. Different countries have different size criteria to classify small hydropower project capacity
ranging from 10MW to 50 MW. In India, hydropower plants of 25 MW or below capacity are
classified as small hydro, which have further been classified into micro (100kW or below), mini
(101kW-2MW) and small hydro (2-25MW) segments.
• The estimated potential of 21135.37 MW from 7135 sites for power generation in the country from small /
mini hydel projects is assessed by the Alternate Hydro Energy Centre (AHEC) of IIT Roorkee in its Small
Hydro Database of July 2016. The hilly States of India mainly Arunachal Pradesh, Himachal Pradesh,
Jammu & Kashmir and Uttarakhand, and constitute around half of this potential. Other potential States
are Maharashtra, Chhattisgarh, Karnataka and Kerala.
• Statement 2 is correct: Karnataka leads in the production of energy from small-hydro power plants.
Karnataka is the first state in the country to set up a professionally managed corporation to plan, construct
operate and maintain power generation projects. The total installed generating capacity of the state has
been 14327.725 MW of which hydro contributed 6498. Karnataka state is endowed with hydropower
potential estimated to be about 7000MW.

Q 44.A
• The National Adaptation Fund for Climate Change (NAFCC) is a Central Sector Scheme which was set
up in the year 2015-16. The overall aim of NAFCC is to support concrete adaptation activities which
mitigate the adverse effects of climate change. Under NAFCC 100% central grant is provided to the
State Governments for implementing climate change adaptation projects. Hence, statement 1 is
correct.
• The activities under this scheme are implemented in a project mode. The projects related to adaptation in
sectors such as agriculture, animal husbandry, water, forestry, tourism etc. are eligible for funding under
NAFCC. National Bank for Agriculture and Rural Development (NABARD) is the National
Implementing Entity (NIE). Hence, statement 2 is not correct.
• Objectives:
o Funding concrete adaptation projects/programmes aligned with the relevant Missions under National
Action Plan on Climate Change (NAPCC) and the State Action on Climate Change (SAPCCs) in
agriculture, horticulture, agroforestry, environment, allied activities, water, forestry, urban, coastal
and low-lying system, disaster management, human health, marine system, tourism, habitat sector and
other rural livelihood sectors to address climate change-related issues.
o Preparing and updating climate scenario, assessing vulnerability and climate impact assessment
o Capacity building of various stakeholders on climate change adaptation and project cycle
management and developing a knowledge network
o Mainstreaming the approaches/ learnings from project/programme implementation through
Knowledge Management.

Q 45.D
• Eutrophication is an enrichment of water by nutrient salts that causes structural changes to the
ecosystem such as increased production of algae and aquatic plants, depletion of fish species,
general deterioration of water quality, and other effects that reduce and preclude use.
• Eutrophication is a serious environmental problem since it results in a deterioration of water quality.
According to the Survey of the State of the World’s Lakes, a project promoted by the International
Lake Environment Committee, eutrophication affects 54% of Asian lakes, 53% of those in Europe, 48%
of those in North America, 41% of those in South America and 28% of those in Africa.

17 www.visionias.in ©Vision IAS


• All water bodies are subject to a natural and slow eutrophication process, which in recent decades has
undergone a very rapid progression due to the presence of man and his activities (so-called cultural
eutrophication). The cultural eutrophication process consists of a continuous increase in the contribution
of nutrients, mainly nitrogen and phosphorus (organic load) until it exceeds the capacity of the water body
(i.e. the capacity of a lake, river, or sea to purify itself), triggering structural changes in the waters. These
structural changes mainly depend on 3 factors:
o Use of fertilizers: Agricultural practices and the use of fertilizers in the soil contribute to the
accumulation of nutrients.
o Discharge of wastewater into water bodies.
o Reduction of self-purification capacity: Over the years, lakes accumulate large quantities of solid
material transported by the water (sediments). These sediments are such as to able to absorb large
amounts of nutrients and pollutants, which ultimately decreases with time.
• Control Measures:
o improvement of the purifying performance of wastewater treatment plants, installing tertiary
treatment systems to reduce nutrient concentrations;
o implementation of effective filter ecosystems to remove nitrogen and phosphorus present in the run-
off water (such as phyto-purification plants);
o reduction of phosphorous in detergents;
o rationalization of agricultural techniques through proper planning of fertilization and use of slow-
release fertilizers like neem coated urea;
o use of alternative practices in animal husbandry to limit the production of wastewater.
• In cases where water quality is already so compromised as to render any preventive initiative ineffective,
“curative” procedures can be implemented, such as:

o removal and treatment of hypolimnetic water (deep water in contact with the sediments) rich in
nutrients since in direct contact with the release source;
o drainage of the first 10-20 cm of sediment subject to biological reactions and with high phosphorus
concentrations;
o oxygenation of water for restoring the ecological conditions, reducing the negative effects of the
eutrophic process, such as scarcity of oxygen and formation of toxic compounds deriving from the
anaerobic metabolism;
o chemical precipitation of phosphorous by the addition of iron or aluminium salts or calcium
carbonate to the water, which gives rise to the precipitation of the respective iron, aluminium or
calcium orthophosphates, thereby reducing the negative effects related to the excessive presence of
phosphorus in the sediments. Hence the correct answer is option (d).

Q 46.B
• National Medicinal Plants Board (NMPB) is an apex national body that coordinates all matters relating
to the medicinal plant sector in the country. It is under the Ministry of AYUSH. Hence statement 1 is not
correct.
• The Board was established in 2000 and acts as an advisory body to the concerned ministries,
departments, and agencies in strategic planning of medicinal plants related initiatives and to plan
and provide financial support to programs relating to conservation, cultivation and also all-round
development of medicinal plants sector. Hence statement 2 is correct.
• NMPB’s main functions include:
o Identification, Inventorisation, and Quantification of medicinal plants.
o Promotion of in-situ / ex-situ conservation and cultivation of medicinal plants
o Undertaking and awarding Scientific, Technological research, and cost-effectiveness studies.
o Development of protocols for cultivation and quality control.
o Encouraging the protection of patent rights and IPR.

Q 47.B
• Recent Context – The research paper published in ‘Indian Pediatrics’, shows that several children (0-18
years) in India recovering from Covid-19 disease show symptoms of Kawasaki disease (KD).
• About Kawasaki Disease
• It is an acute inflammatory disease of the blood vessels and usually occurs in children below the age of
five.
• The disease was first described in Japan by Tomisaku Kawasaki in 1967.

18 www.visionias.in ©Vision IAS


• Causing factors- Scientists haven’t found an exact cause for Kawasaki disease. It might be linked to
genes, viruses, bacteria, and other things in the world around a child, such as chemicals and
irritants. Hence statement 1 is not correct.
• Symptoms- Clinical signs include fever, rash, swelling of the hands and feet, irritation and redness of the
whites of the eyes, swollen lymph glands in the neck, and irritation and inflammation of the mouth, lips,
bilateral conjunctival hyperemia, mucosal changes of the oropharynx, erythematous rash, erythema and
throat. The inflammation caused by the disease affects many parts of the body but has a more serious
effect on the heart since it causes inflammation in the coronary arteries that are responsible for supplying
blood to the heart. This results in enlargement or in the formation of aneurysms that can lead to heart
attacks. Hence statement 2 is correct.
• Treatment- The standard treatment, intravenous immunoglobulin and aspirin, substantially decreases the
development of these coronary artery abnormalities.
• Recent observation -In the recovery phase of the COVID 19 disease, less than 1 per cent in the world
over have shown to be affected by either Kawasaki Disease (KD) or Multisystem Inflammatory Syndrome
in Children (MIS-C).
• KD or MIS-C are the results of a ‘cytokine storm’ in children, usually after one month of recovery from
COVID-19. Cytokines are produced in the body in response to an infection to fight against the virus.
Cytokine storm is one of the major causes of acute respiratory distress syndrome, multi-organ dysfunction
and possibly pediatric multisystem inflammatory syndrome.

Q 48.D
• The Bharat Stage emission standards are standards instituted by the government to regulate the output
of air pollutants from motor vehicles from internal combustion engine equipment, including motor
vehicles. India has been following European (Euro) emission norms, although with a time lag of five
years.
• In 2016, the government said India would directly progress from BS-IV norms to BS-VI, skipping the
intermediary stage of BS-V. In April 2020, India moved towards more stringent BS-VI norms. Hence
statement 1 is not correct.
• The main difference in standards between the existing BS-IV and the new BS-VI auto fuel norms is the
presence of sulphur. The BS-VI fuel is estimated to bring around an 80 per cent reduction of sulphur,
from 50 parts per million to 10 ppm. Hence statement 3 is not correct.
• While the difference in emission levels is not that drastic for petrol engines, it is significant for diesel
vehicles. For example, the nitrous oxide emission levels (responsible for acid rain) will drop by 25
percent in the case of petrol vehicles and 68 percent for diesel vehicles. Hence getting a BS-IV diesel
engine to comply with BS-VI emission norms requires major changes to the hardware and layout of a
diesel engine compared to a petrol engine. That is why many car manufacturers in India are already
rolling out BS-VI compliant petrol models, but holding back on diesel models. Hence statement 2 is not
correct.
• The new technologies needed to be included in the vehicles include Diesel Particulate filter to expel
particulate matters, Selective catalytic reduction and exhaust gas regulator to reduce NOx emissions.

Q 49.A
• Biological Diversity Act 2002 mandates all local bodies to set up Biodiversity Management
Committees (BMC). As per the Act, the local bodies constitute the BMC in accordance with Section 41,
within their area of jurisdiction for the purpose of promoting conservation, sustainable use and
documentation of biological diversity including preservation of habitats, conservation of landraces, folk
varieties and cultivars, domesticated stocks and breeds of animals, micro-organisms and chronicling of
knowledge relating to biological diversity.
• The BMC consists of a Chairperson, and six persons nominated by local bodies, including 1/3rd women
and 18% SC/ST.
• The functions of BMCs are as under:
o Prepare, maintain and validate People’s Biodiversity Register (PBR) in consultation with the
local people. The BMC is to maintain a Register giving information about the details of biological
resources and traditional knowledge available within the jurisdiction of BMC.
o Advice on any matter referred to it by the State Biodiversity Board or Authority for granting approval,
to maintain data about the local vaidhya (traditional healer) and practitioners using the biological
resources.
o Preserve and promote local biodiversity- breeds of birds, animals and plants i.e. preservation of
habitats.
19 www.visionias.in ©Vision IAS
o Advice State & National Biodiversity Boards on matters related to local biodiversity.
o Under the Nagoya Protocol of Convention on Biodiversity (CBD), they can collect fees for granting
access to Biodiversity register to researchers and commercial companies.
• Thus in a nutshell, the Local Level Biodiversity Management committees (BMCs) are responsible for
promoting conservation, sustainable use and documentation of biological diversity including
preservation of habitats, conservation of landraces, folk varieties and cultivators, domesticated stocks
and breeds of animals and microorganisms besides chronicling of knowledge relating to biological
diversity. Hence, options 1 and 2 are correct.
• The State Biodiversity Board (SBBs) regulate by granting of approvals or otherwise upon requests for
commercial utilization or bio-survey and bio-utilization of any biological resource by the Indians and not
the Biodiversity Management committees Hence option 3 is not correct.

Q 50.D
• Reducing emissions from deforestation and forest degradation (REDD+) is a mechanism developed
by Parties to the United Nations Framework Convention on Climate Change (UNFCCC).
• Its objective is to create a financial value for the carbon stored in forests by offering incentives for
developing countries to reduce emissions from forested lands and invest in low-carbon paths to
sustainable development
• Developing countries would receive results-based payments for results-based actions. REDD+ goes
beyond simply deforestation and forest degradation and includes the role of conservation, sustainable
management of forests and enhancement of forest carbon stocks. REDD+ aims to achieve climate
change mitigation by incentivizing forest conservation.
• Complying with the UNFCCC decisions on REDD+, India has prepared its National REDD+
Strategy. The Strategy builds upon existing national circumstances which have been updated in line with
India’s National Action Plan on Climate Change, Green India Mission and India’s Nationally Determined
Contribution (NDC) to UNFCCC.
• Hence option (d) is the correct answer.

Q 51.B
• The International Renewable Energy Agency is an intergovernmental organisation mandated to
facilitate cooperation, advance knowledge, and promote the adoption and sustainable use of renewable
energy.
• India is one of the Founding Members of the International Renewable Energy Agency (IRENA),
which is an intergovernmental organization that supports countries in their transition to a sustainable
energy future and serves as the principal platform for international cooperation in Renewable Energy, a
centre of excellence and a repository of knowledge in policy matters, RE technologies, RE resources and
financing of RE sector. IRENA promotes adoption and sustainable use of all forms of renewable energy,
including bioenergy, geothermal, hydropower, ocean, solar and wind energy in the pursuit of sustainable
development, energy access, energy security and low-carbon economic growth and prosperity. Hence the
correct answer is option (b).

Q 52.C
• Radon (Rn) is a noble gas produced by the radioactive decay of radium; it is a decay product of
uranium and thorium, which occur naturally in Earth's crust. It is an invisible, odorless, and tasteless
gas. Radon sinks in air because it has a high density; it is therefore often found in the basements of
homes, particularly in areas where with a lot of shale and boulders in the soil. Hence statement 1 is
correct and statement 2 is not correct.
• Because it is inert, radon itself does not pose a hazard. However, it undergoes radioactive decay
producing a series of short-lived progeny, often called daughters, that can emit alpha, beta, or gamma
particles and are electrically charged, readily attaching to air-borne particles.
• Outdoors, radon quickly dilutes to very low concentrations and is generally not a problem.
However, indoors, radon concentrations are higher, with highest levels found in places like mines, caves
and water treatment facilities. In buildings such as homes, schools, and offices radon levels are found to
be high in especially those constructed with phosphate rocks and other radon emitting rocks. Radon gas
can also dissolve and accumulate in water from underground sources (ground water), such as wells.
When water that contains radon is used in the home for showering, washing dishes, and cooking, radon
gas escapes from the water and goes into the air. Hence statement 3 is correct.

20 www.visionias.in ©Vision IAS


• Radon is a gas, but these decay products are solids that can attach to dust and enter the lungs. Radon and
its daughters continue to decay in the lungs, releasing alpha and beta particles that can damage cellular
DNA and result in lung cancer. Radon and its daughters are the leading cause of lung cancer in non-
smokers.
• Radon levels in existing homes can be reduced by: sealing floors and walls; and improving the ventilation
of the house etc.

Q 53.A
• The Global Environment Outlook (GEO) is the UN Environment Programme’s (UNEP) flagship
environmental assessment. The first publication was in 1997 and was originally requested by the
Member States. The Global Environment Outlook Report 2019 is the sixth edition. Hence option (a) is
the correct answer.
• It outlines the current state of the environment that illustrates possible future environmental trends and
analyses the effectiveness of policies.
• Key Findings of the GEO-6 report
o As greenhouse gas emissions continue to rise amid an increased risk of droughts, floods and
superstorms made worse by climbing sea levels, there is a growing political consensus that climate
change poses a future risk to billions.
o Lacking access to clean drinking supplies, 1.4 million people die each year from preventable diseases
such as diarrhoea and parasites linked to pathogen-riddled water and poor sanitation.
o Chemicals pumped into the seas cause “potentially multi-generational” adverse health effects.
o The report called for a decrease in greenhouse gas emissions and pesticide use to improve air and
water quality.
o Food waste for instance, which accounts for 9% of global greenhouse gas emissions, could be slashed.
o The world currently throws away a third of all food produced. In richer nations, 56% goes to waste.
o The report advises adopting less-meat intensive diets and reducing food waste in both developed and
developing countries.

Q 54.A
• Ozone is natural gas. It is an allotrope of oxygen. It is found in two different layers of the
atmosphere. Ozone in the troposphere is “bad” because it dirties the air and helps to form
photochemical smog. Ozone in the stratosphere is “good” because it protects life on Earth by absorbing
some of the sun’s harmful Ultra Violet (UV) rays. It helps in reducing the risks of mutation and harm to
plant and animal life.
• Ozone depletion refers to the gradual thinning of Earth’s ozone layer in the upper atmosphere caused by
the release of chemical compounds containing gaseous chlorine or bromine from industry and other
human activities.
• Principal steps in stratospheric ozone depletion:
o The stratospheric ozone depletion process begins with the emission of halogen source gases by
human activities and natural processes. These compounds have at least one carbon and one halogen
atom, causing them to be chemically stable.
o Accumulation: Halogen source gases accumulate in the atmosphere and are globally distributed
throughout the lower atmosphere by winds and other air motions.
o Transport: Halogen source gases are transported to the stratosphere by air motions.
o Conversion: Most halogen source gases are converted in the stratosphere to reactive halogen gases in
chemical reactions involving ultraviolet radiation from the sun.
o Chemical reaction: Reactive halogen gases cause chemical depletion of stratospheric ozone over the
globe. Low-temperature surface reactions on polar stratospheric clouds (PSCs) significantly
increase reactive halogen gases and thereby cause severe ozone loss in polar regions in late winter and
early spring.
• The Dobson Unit is the most common unit for measuring ozone concentration. One Dobson Unit is the
number of molecules of ozone that would be required to create a layer of pure ozone 0.01 millimeters
thick at a temperature of 0 degrees Celsius and a pressure of 1 atmosphere (the air pressure at the surface
of the Earth). The average amount of ozone in the atmosphere is roughly 300 Dobson Units. What
scientists call the Antarctic Ozone “Hole” is an area where the ozone concentration drops to an average
of about 100 Dobson Units.
• Vienna convention adopted in the year 1985 and entered into force in 1988. The Vienna Convention for
the Protection of the Ozone Layer and its Montreal Protocol on Substances that Deplete the Ozone Layer
are dedicated to the protection of the earth’s ozone layer.
• Hence option (a) is the correct answer.

21 www.visionias.in ©Vision IAS


Q 55.D
• Project Elephant was launched in 1992 by the Government of India Ministry of Environment and
Forests to provide financial and technical support to wildlife management efforts by states for their
free-ranging populations of wild Asian Elephants.
• The project aims to ensure the long-term survival to the populations of elephants in their natural habitats
by protecting the elephants, their habitats and migration corridors. Other goals of Project Elephant are
supporting the research of the ecology and management of elephants, creating awareness of conservation
among local people, providing improved veterinary care for captive elephants.
• Project Elephant (PE) was launched as a Centrally Sponsored Scheme with the following objectives:
o To protect elephants, their habitat & corridors.
o To address issues of man-animal conflict.
o The welfare of captive elephants. Hence option (d) is the correct answer.
• As a part of the project following activities are being conducted:
o Financial support is being provided to the major elephant bearing States in the country. The Project is
being mainly implemented in 16 States / UTs, viz. Andhra Pradesh, Arunachal Pradesh, Assam,
Chhattisgarh, Jharkhand, Karnataka, Kerala, Maharashtra, Meghalaya, Nagaland, Orissa, Tamil Nadu,
Tripura, Uttarakhand, Uttar Pradesh, West Bengal.
o Ecological restoration of existing natural habitats and migratory routes of elephants are built.
o Development of scientific and planned management for conservation of elephant habitats and viable
population of Wild Asiatic elephants in India;
o Promotion of measures for mitigation of man-elephant conflict in crucial habitats and moderating
pressures of human and domestic stock activities in crucial elephant habitats;
o Strengthening of measures for the protection of Wild elephants from poachers and unnatural causes of
death;
o Research on Elephant management related issues;
o Public education and awareness programmes;
o Eco-development
o Veterinary care
o Elephant Rehabilitation/Rescue Centers
• As of 2020, 32 Elephant Reserves (ERs) extending over about 58,000 square kilometres (22,000 sq mi)
have been formally notified by various State Governments.

Q 56.D
• Recent Context - Recently, Willow Warbler was seen for the first time in the Vellayani-Punchakkari rice
fields in Kerala, India. Hence statement 1 is not correct.
• About Willow Warbler
o It is one of the longest migrating small birds that breeds throughout northern and temperate Europe
and the Palearctic.
o It is peculiar as while it weighs around 10 grams, its long wing feathers help fly long distances.
o It is usually seen in the European and the Palearctic regions. It migrates to sub-Saharan Africa during
early winter.
o Warblers are generally difficult to identify owing to the small size and change in plumage twice a
year.
o It is listed as Least Concern under the IUCN Red List of threatened species. Hence statement 2
is not correct.

Q 57.D
• Society of Integrated Coastal Management (SICOM) has been established under the aegis of the
Ministry of Environment, Forests and Climate change, Government of India with a vision for
vibrant, healthy and resilient Coastal and Marine Environment for the continuous and enhanced outflow
of benefits to the Country and the Coastal Community. Hence statement 3 is correct.
• Ministry of Environment and Forests is implementing the World Bank assisted Integrated Coastal Zone
Management (ICZM) project through the Society of Integrated Coastal Management (SICOM). SICOM
is the National Project Management Unit of the ICZM project. SICOM is located in Delhi and
operating in all the coastal states and Union Territories of Government of India. Hence statement 1 is
correct.

22 www.visionias.in ©Vision IAS


• Objective and Functions of SICOM
o To support the implementation of the Integrated Coastal Zone Management of India.
o To promote Research & Development (R&D) and stakeholder participation in the management of the
Coastal areas of India.
o To support to check violations to CRZ (Coastal Regulation Zone) through improved
technology-enabled enforcement strengthened institutions and regulatory and legal reforms.
Hence statement 2 is correct.
o To undertake any additional work or function as may be assigned by MOEF&CC from time to time in
the areas of coastal management and other related activities
• SICOM has also embarked upon the Pilot Blue Flag Programme, a first in India under Beach Environment
& Aesthetic Management Services (BEAMS) for the development of world-class Beaches in India.
Capacity building of Coastal State Units in planning, implementation and management of Integrated
Coastal Zone Management programs and other national and State organization/agencies associated with
the project serving as an interface among coastal communities, experts, and governments, including
providing with and disseminating examples of best practices and guidelines for coastal and marine
ecological security and livelihood security of coastal and island communities.
• Hence option (d) is the correct answer.

Q 58.A
• The Solar City aims at a minimum of 10% reduction in projected demand of conventional energy at
the end of five years, through a combination of enhancing supply from renewable energy sources in the
city and energy efficiency measures.
• The basic aim is to motivate the local Governments for adopting renewable energy technologies and
energy efficiency measures.
• In a Solar City, all types of renewable energy-based projects like solar, wind, biomass, small hydro, waste
to energy etc. may be installed along with possible energy efficiency measures depending on the need and
resource availability in the city.
• The city is identified based on city population, potential and commitment for the adoption of
renewable energy and energy conservation in the city activities, initiatives are already taken by Local
Bodies/ Private Developers/Industry/General Public in promoting renewable energy and energy
conservation, regulatory measures taken on the deployment of renewable energy technologies and their
willingness to provide resources and sustenance of activities initiated under the program.
• The cities may have a population between 0.50 lakh to 50 lakh, however, relaxation could be
considered for special category States including the North-Eastern States and hilly States, Islands and
Union Territories.
• Objectives of the Solar City programme
o To enable and empower Urban Local Governments to address energy challenges at City - level.
o To provide a framework and support to prepare a Master Plan including assessment of the current
energy situation, future demand and action plans.
o To build capacity in the Urban Local Bodies and create awareness among all sections of civil society.
o To involve various stakeholders in the planning process.
o To oversee the implementation of sustainable energy options through public-private
partnerships. Hence option (a) is the correct answer.

Q 59.C
• Both statement 1 and statement 2 are correct: National Institute of Solar Energy, an autonomous
institution of Ministry of New and Renewable (MNRE), is the apex National R&D institution in the field
Solar Energy. The Government of India has converted 25 year old Solar Energy Centre (SEC) under
MNRE to an autonomous institution in September, 2013 to assist the Ministry in implementing the
National Solar Mission and to coordinate research, technology and other related works.
• Under the skill development initiatives of the MNRE, NISE is mandated as the nodal agency for
implementation of “Suryamitra Skill Development Programme (SSDP)”. SSDP aims to develop the skills
of youth, considering the opportunities for employment in the growing Solar Energy Power Project’s
installation, operation & maintenance in India and abroad. The SSDP is also designed to prepare the
candidates to become new entrepreneurs in Solar Energy sector.
23 www.visionias.in ©Vision IAS
Q 60.B
• Environment (Protection) Act:
• The genesis of the Environmental (Protection) Act, 1986 is in Article 48A (Directive Principles of State
Policy) and Article 51A (Fundamental Duties) of the Indian Constitution.
• The act authorizes the central government to protect and improve environmental quality, control and
reduce pollution from all sources, and prohibit or restrict the setting or operation of any industrial facility
on environmental grounds.
• The act prescribes special procedure for handling hazardous substances.
• The act grants immunity to the officers of the Government for any act done under the provisions of this
Act.
• The Act debars the Civil Courts from having any jurisdiction to entertain any suit or proceeding in
respect of an action under the Act.
• The rules or orders made under this Act shall have supremacy over anything inconsistent contained in
any enactment other than this Act.
• Important Government bodies related to Environment and Pollution:
o The Central Pollution Control Board (CPCB) is a statutory organisation constituted under
the Water (Prevention and Control of Pollution) Act, 1974. Further, CPCB was entrusted with the
powers and functions under the Air (Prevention and Control of Pollution) Act, 1981. Principal
Functions of the CPCB are to promote cleanliness of streams and wells by prevention and control of
water pollution, and to improve the quality of air.
o Central Ground Water Authority has been constituted under the Environment (Protection) Act,
1986 to regulate and control development and management of ground water resources in the country.
CGWA has been regulating ground water development through measures such as issue of advisories,
public notices, grant of No Objection Certificates (NOC) for ground water withdrawal.
o National Tiger Conservation Authority (NTCA) is a statutory body constituted under provisions of
the Wildlife (Protection) Act, 1972 for strengthening tiger conservation, as per powers and functions
assigned to it. It was established in 2005 following the recommendations of the Tiger Task Force.
o The Environment Pollution (Prevention and Control) Authority (EPCA) was constituted under
the provisions of the Environment (Protection) Act, 1986 with the aim of protecting and improving
the quality of the environment and preventing and controlling the environmental pollution in the
National Capital Region, The EPCA is mandated to enforce Graded Response Action Plan (GRAP)
in the city as per the pollution levels.
o National Ganga River Basin Authority(NGRBA) was constituted under the provisions of
the Environment (Protection) Act,1986. NGRBA has been dissolved with effect from October 2016,
consequent to constitution of National Council for Rejuvenation, Protection and Management of
River Ganga (referred as National Ganga Council) under the Act. The National Ganga Council is
chaired by Prime Minister. The Council has been given overall responsibility for superintendency of
pollution prevention and rejuvenation of River Ganga Basin.
• Hence option (b) is the correct answer.

Q 61.D
• Ammonia (NH3) is a toxic gas with a pungent smell that irritates the eyes and respiratory system. Once
emitted, ammonia reacts quickly with other air pollutants to form ammonium sulfate and ammonium
nitrate, contributing to the overall particulate matter burden. However, ammonia and its subsequent
particulate ammonium salts are important sources of nitrogen in soils and contribute to plant
growth. The use of ammonia-based synthetic fertilisers has boosted agricultural productivity, which in
turn has led to dramatic increases in emissions of ammonia.
• The sources of ammonia emission include farming, animal husbandry and its associated manure
processes, industrial processes, combustion processes, biomass burning, denitrification units installed to
reduce nitrogen oxides emissions from power stations, and catalytic converters in motor vehicles.
Hence option (d) is the correct answer.
• Under typical ambient conditions, ammonia (NH3) reacts with other inorganic species, in particular
sulfuric acid (H2SO4-) and nitric acid (HNO3), to form salts. Ammonium nitrate, originating from the
reaction of ammonia with nitric acid, is a main source of fine particulate nitrate. Ammonium sulfate
and ammonium nitrate remain in the atmosphere as a particulate matter on the order of days and for up
to a week.
• Ammonia decreases the biodiversity of terrestrial and aquatic ecosystems and also forms aerosols in the
atmosphere which can cause human health complications if inhaled.

24 www.visionias.in ©Vision IAS


• Ammonia pollution regulations mainly focus on mitigation through better farming practices. These
include
o Implementing floating covers such as plastic sheeting or ‘low-technology’ floating covers
o Allowing a natural crust to form on the surface of the manure slurry
o keeping manure and fertilizer in large storage tanks to prevent runoff and volatilization into the air
o feeding livestock diets less dense in protein
o Using less urea and ammonium based fertilizers which are prone to volatilization into ammonia.

Q 62.C
• Section 2 (b) Schedule Tribes and Other Traditional Forest Dwellers (Recognition of Forest Rights)
Act, 2006, provides for determination and notification of 'Critical Wildlife Habitats' within
National Parks and Sanctuaries.
• The act defines 'Critical Wildlife Habitat ' as such areas of National Parks and Sanctuaries where it has
been specifically and clearly established, case by case, on the basis of scientific and objective criteria, that
such areas are required to be kept as inviolate for the purpose of wildlife conservation.
• As per the sec 2(b) of FRA, 2006, the Ministry of Environment and Forests (MoEF) has been
identified as the agency to determine and notify Critical Wildlife Habitats. The Ministry of Tribal
Affairs (MoTA) is the nodal ministry for recognition and vesting of individual and community forest
rights.
• Hence, both statements are not correct.

Q 63.A
• Any unwanted sound that causes annoyance, irritation and pain to the human ear is termed noise. Earlier,
noise pollution and its sources were addressed under the Air (Prevention and Control of Pollution) Act,
1981. They are now, however, regulated separately under the Noise Pollution (Regulation and Control)
Rules, 2000 which are formulated under the provisions of the Environment (Protection) Act, 1986.
Hence statement 1 is not correct.
• The Central Pollution Control Board has laid down the permissible noise levels in India for different
areas. Noise pollution rules have defined the acceptable level of noise in different zones for both daytime
and night time. Hence statement 2 is correct.
• In industrial areas, the permissible limit is 75 dB for daytime and 70 dB at night. In commercial
areas, it is 65 dB and 55 dB, while in residential areas it is 55 dB and 45 dB during daytime and night
respectively. Additionally, state governments have declared ‘silent zones’ which includes areas that lie
within 100 meters of the premises of schools, colleges, hospitals and courts. The permissible noise limit in
this zone is 50 dB during the day and 40 dB during the night. Hence statement 3 is not correct.
• Recently the Central Pollution Control Board (CPCB) has proposed a new set of fines between Rs
1,000 and Rs 1 lakh for those who violate norms restricting noise pollution under the Noise Pollution
(Regulation and Control) Rules, 2000.

Q 64.B
• Recent Context - Astronomers have revealed that a large asteroid named ‘Apophis’ is expected to pass
extremely close or may hit the Earth in 2068 due to a phenomenon called the Yarkovsky effect.
• About Yarkovsky effect
o It is the force exerted on an object when sunlight is absorbed and released. Hence option (b) is the
correct answer.
o When sunlight strikes a rotating asteroid like Apophis, the dayside heats up; as the asteroid turns, the
night side cools and releases the heat. Under certain circumstances, the sun can heat an asteroid
unevenly, causing the space rock to radiate away heat energy asymmetrically. The result can be a tiny
push in a certain direction which is called Yarkovsky acceleration. It can change the path of an
asteroid through space.
• About asteroid Apophis
o Asteroids are small, rocky objects that orbit the Sun. Although asteroids orbit the Sun like planets,
they are much smaller than planets.
o Asteroid Apophis is equivalent to the size of three football grounds.
o It was discovered in 2004. It was earlier predicted to pass really close to the earth in 2068. However,
scientists with new calculations have said that its collision with the earth cannot be ruled out
altogether. The asteroid orbit has been disturbed by the Yarkovsky effect.

25 www.visionias.in ©Vision IAS


Q 65.B
• Recent Context - Pilibhit Tiger Reserve (PTR) and the Uttar Pradesh Forest department have bagged the
first-ever international award, TX2 for doubling the number of tigers in four years (25 in 2014 to 65 in
2018) against a target of 10 years. PTR was the first to receive the award among 13 tiger range countries.
• About TX2 Program:
o The TX2 goal is a global commitment to double the world's wild tigers by 2022 from the 2010 levels.
o TX2 stands for 'Tigers times two', signaling the goal to double wild tigers.
o It is an initiative of the World-Wide Fund for Nature (WWF) launched at the 2010 St Petersburg Tiger
Summit.
o Under this program, all 13 tiger range governments have committed to double the number of wild
tigers by 2022.
o 13 tiger range countries: India, Bangladesh, Bhutan, Cambodia, China, Indonesia, Lao PDR,
Malaysia, Myanmar, Nepal, Russia, Thailand and Vietnam.
• About Pilibhit Tiger Reserve
o It is located in Uttar Pradesh.
o The northern edge of the reserve lies along the Indo-Nepal border while the southern boundary is
marked by the river Sharada and Khakra.
o It is the 45th tiger reserve project in India.
• Hence option (b) is the correct answer.

Q 66.C
• Recent Context: The INTERPOL and the World Customs Organization conducted Operation
Thunder, 2020 in 103 countries. The operation was conducted against environmental crimes. Hence
option (c) is the correct answer.
o In India, the India Customs along with INTERPOL and WCO seized 18 tonnes of red sandalwood that
were destined for UAE.
• Operation Thunder is being held since 2017. Under the operation, the participating countries focused on
particularly vulnerable species protected under CITES (Convention on International Trade in Endangered
Species in Wild Fauna and Flora).
• About INTERPOL: INTERPOL was established in 1923 as the International Criminal Police
Organisation (ICPO), much before the formation of the United Nations.
o The INTERPOL organization incorporates 194 member countries and has observer status in the
United Nations.
o The founding members include police officers from Austria, Germany, Belgium, Poland, China,
Egypt, France, Greece, Hungary, Italy, the Netherlands, Romania, Sweden, Switzerland, and
Yugoslavia. India is not one of its founding members.
• About World Customs Organization: The World Customs Organization (WCO) is an
intergovernmental organization headquartered in Brussels, Belgium. It was established in 1952. It has 183
members. India though a member but not a founding member.
o The WCO’s primary objective is to enhance the efficiency and effectiveness of member customs
administrations, thereby assisting them to contribute successfully to national development goals,
particularly revenue collection, national security, trade facilitation, community protection, and
collection of trade statistics.
o The WCO is noted for its work in areas covering the development of international conventions,
instruments, and tools on topics such as commodity classification, valuation, rules of origin,
combating counterfeiting in support of Intellectual Property Rights (IPR), drugs enforcement, illegal
weapons trading, etc.
o The WCO maintains the international Harmonized System (HS) goods nomenclature and administers
the technical aspects of the World Trade Organization (WTO) Agreements on Customs Valuation and
Rules of Origin.

Q 67.D
• Wildlife protection act,1972
• Special plants are those plants that are mentioned in Schedule VI of wildlife protection act,1972
o Cycas beddomi
o Red vanda
26 www.visionias.in ©Vision IAS
o Blue vanda
o Pitcher plant (Sasuaria lappa)
o Ladies slipper orchid
• Lantana camara is a small perennial shrub, which forms extensive, dense and impenetrable thickets. It
is native to Central and South America. It is a invasive species. Hence option d is correct.

Q 68.A
• Recent Context - RT-LAMP (Reverse Transcriptase loop-mediated isothermal amplification) technology
is now being used in India for COVID-19 diagnosis.
• About RT-PCR
o RT-PCR stands for Real-Time Reverse Transcription Polymerase Chain Reaction.
o It is used for the detection of diagnosis of novel coronavirus – SARS-CoV-2, responsible for Covid-
19 disease.
o It detects the presence of viral nucleic acids in nasopharyngeal swab samples.
• Disadvantages of RT-PCR
o It requires complex and costly equipment.
▪ The test needs different temperatures in one cycle. The temperature of the solution has to be
changed from 92 degrees C to 56 degrees C and again to 72 degrees C every two minutes, and this
cycle has to be repeated. Thus, the PCR test needs an expensive thermal cycler as well as real-
time PCR machines.
▪ This test requires reagents to be stored and transported at –20 degrees C, which needs deep
freezers that escalate the cost
o It requires extensive training for potential users.
o As the specificity and sensitivity of the test is about 95%, there is a possibility of false-negative
results.
o The turnaround time is about 10 hours so that the result will be available only by the next day. In
remote places, the turnaround time further increases depending on the distance the samples need to
travel.
• About RT-LAMP
o RT-LAMP stands for Reverse Transcriptase loop-mediated isothermal amplification.
o It is now being used in India to detect novel coronavirus – SARS-CoV-2.
o It is a one-step nucleic acid amplification method to multiply specific sequences of RNA of the
coronavirus. Here, the RNA is first made into cDNA (copy DNA) by the usual reverse transcription.
Then, the DNA is amplified by the LAMP technique.
• RT-LAMP technology has many advantages over RT-PCR technology.
o LAMP is cost-effective and does not need complex expensive equipment and can be set up at
mobile units/kiosks for testing at Airports, Railway Stations, Bus Stands, and other public
places. Whereas, RT-PCR which is expensive; requires highly trained manpower, costly
instruments, and a relatively high-end lab and cannot be deployed at remote locations in quarantine
centers, airports, and railway stations, etc. Hence statement 2 is not correct.
o The RT-PCR test needs different temperatures in one cycle. The temperature of the solution has to
be changed from 92 degrees C to 56 degrees C and again to 72 degrees C every two minutes, and this
cycle has to be repeated. Thus, the PCR test needs an expensive thermal cycler as well as real time
PCR machines. On the other hand, the new RT-LAMP technology is done at 65 degrees C, where
the DNA amplification is done at a constant temperature (isothermal), so an expensive thermal
cycler is not required.
o The quantity of DNA amplified in the LAMP technology is a hundred thousand times more than that
is taking place in PCR. Therefore the final assay is possible with a simple color reaction, removing the
need for very costly real-time PCR machines.
o Another advantage of LAMP is that the reagents are to be stored at 4 degrees C (ordinary fridge),
whereas the PCR-based reagents are to be stored and transported at –20 degrees C, which
needs deep freezers that escalate the cost.
o An equally important advantage of the LAMP technology is that the assay is so fast that results
can be obtained within 30 minutes and positive samples are amplified as early as 10
minutes. This may be compared with the PCR technique which needs 8–10 hours for completion.
This means that COVID-19 testing centers can report with accuracy in substantially lower turnover
time. Hence statement 1 is correct.

27 www.visionias.in ©Vision IAS


Q 69.C
• Recent Context - The isolated UK Overseas Territory of Tristan da Cunha which is home to the
world’s most remote human settlement recently declared as the largest fully protected marine reserves
in the Atlantic Ocean.
• About Tristan da Cunha:
o It is a small chain of islands over 6,000 miles from London in the South Atlantic and the water around
the islands is considered to be the richest in the world. Hence option (c) is the correct answer.
o It is home to several species of land birds that live nowhere else, including the Wilkins bunting, the
U.K.’s rarest bird, and the Inaccessible Rail, the world’s smallest flightless bird.
o It is also home to the World Heritage Site of Gough and Inaccessible Islands, which is one of the most
important seabird islands in the world.
o According to the Pew Bertarelli Ocean Legacy Project, the island supports 85% of the endangered
northern rockhopper penguins, 11 species of whales and dolphins, and most of the world’s sub-
Antarctic fur seals.
o The island will become the largest no-take zone in the Atlantic Ocean after joining the Blue Belt
program of the UK.

• The Blue Belt Programme was launched in 2017 to protect the overseas territories of the country. This
includes British Antarctic Territories and British Indian Ocean Territories as well.

Q 70.A
• Recent Context – Recently, Ethiopia’s Prime Minister started a military operation in its northern region
of Tigray. The Ethiopian government has said it would be a limited campaign focusing on the Tigrayan
People’s Liberation Front (TPLF), the militia-cum-political party that runs the region.
• About the Conflict:
• The Tigray region is one of 10 semi-autonomous federal states organized along ethnic lines in Ethiopia
and home mostly to the Tigrayan people who make up about 6 percent of Ethiopia’s population of more
than 110 million. Hence option (a) is the correct answer.
• Tigrayans have long been a center of power and influence, controlling the country’s government for three
decades.
• Tigray People’s Liberation Front (TPLF) dominated the ruling alliance composed of four ethnic-regional
parties, until Prime Minister Abiy Ahmed, a member of the Oromo ethnic group, came to power in 2018.
The fortunes of the TPLF have waned since Abiy came to power in 2018.
• Under Abiy, Tigray’s leaders have complained of being unfairly targeted in corruption prosecutions,
removed from top positions, and broadly scapegoated for the country’s woes. A year ago, the TPLF
withdrew from the ruling coalition after Abiy merged it into the nationwide Prosperity Party. The feud
became more intense after Tigray held its own elections in September, defying Abiy’s government which
postponed national polls due to the coronavirus pandemic. Addis Ababa ruled the Tigray government was
unlawful and, in return, Tigray said it no longer recognized Abiy’s administration.
• The federal government then slashed funding to the region, which the TPLF said was “tantamount to an
act of war”. In what the International Crisis Group termed a “sudden and predictable” descent into
conflict, Abiy said on November 4, 2020 that the TPLF had crossed a “red line” and attacked a federal
military base in Tigray, forcing a “military confrontation”. The TPLF accuses Abiy of concocting the

28 www.visionias.in ©Vision IAS


story to justify deploying the military against it. Given Tigray has a powerful military, with an estimated
250,000 troops, a war could be lengthy and bloody.
• There are concerns a conflict in Ethiopia could reverberate across the already fragile Horn of Africa,
impacting neighbors Somalia, Eritrea, Djibouti, and Sudan.

Q 71.B
• Ocean acidification refers to a reduction in the pH of the ocean over an extended period of time,
caused primarily by the uptake of carbon dioxide (CO2) from the atmosphere.
• In chemistry, pH is a scale used to specify the acidity or basicity of an aqueous solution. Acidic solutions
(solutions with higher concentrations of H+ ions) are measured to have lower pH values than basic
or alkaline solutions. Hence statement 1 is not correct.
• When CO2 is absorbed by seawater, a series of chemical reactions occur resulting in the increased
concentration of hydrogen ions. This increase causes the seawater to become more acidic and causes
carbonate ions to be relatively less abundant. Hence statement 2 is correct.
• Carbonate ions are an important building block of structures such as seashells and coral skeletons.
Decreases in carbonate ions can make building and maintaining shells and other calcium carbonate
structures difficult for calcifying organisms such as oysters, clams, sea urchins, shallow water corals,
deep-sea corals, and calcareous plankton.
• These changes in ocean chemistry can affect the behavior of non-calcifying organisms as well. Certain
fish's ability to detect predators is decreased in more acidic waters. When these organisms are at risk, the
entire food web may also be at risk.

Q 72.B
• Bioremediation is the use of microorganisms (bacteria and fungi) to degrade environmental
contaminants into less toxic forms.
• The microorganisms may be indigenous to a contaminated area or they may be isolated from elsewhere
and brought to the contaminated site. The process of bioremediation can be monitored indirectly by
measuring the Oxidation Reduction Potential or redox in soil and groundwater, together with pH,
temperature, oxygen content, electron acceptor/ donor concentrations, and concentration of breakdown
products.
• Bioremediation Strategies:
• In situ bioremediation techniques: It involves treatment of the contaminated material at the site.
o Bioventing: It involves the supply of air and nutrients through wells to contaminated soil to stimulate
the growth of indigenous bacteria. It is used for simple hydrocarbons and can be used where the
contamination is deep under the surface.
o Biosparging: It refers to the Injection of air under pressure below the water table to increase
groundwater oxygen concentrations and enhance the rate of biological degradation of contaminants by
naturally occurring bacteria.
o Bioslurping: It combines approaches of bioventing and vacuum-enhanced free-product recovery to
address two separate contaminant media. Bioventing stimulates aerobic bioremediation of
hydrocarbon-contaminated soils. Vacuum-enhanced free-product recovery extracts groundwater
contaminants to be lifted off the water table and released from the capillary range.
29 www.visionias.in ©Vision IAS
o Bioaugmentation : In this process, Microorganisms are imported to a contaminated site to enhance
the degradation process. Hence option ( b ) is the correct answer.
• Ex -situ Bioremediation: It involves the removal of the contaminated material to be treated elsewhere ,

o Landfarming - contaminated soil is excavated and spread over a prepared bed and periodically tilled
( until pollutants are degraded ). The goal is to stimulate indigenous biodegradative microorganisms
and facilitate their aerobic degradation of contaminants ,

o Biopiles : it is a hybrid of landfarming and composting. Essentially, engineered cells are constructed
as aerated composted piles. Typically used for the treatment of surface contamination with petroleum
hydrocarbons .
o Bioreactors - it involves the processing of contaminated solid material ( soil, sediment, sludge) or
water through an engineered containment system.

Q 73 .C
• The Global Climate Risk Index 2020 by NGO Germanwatch, analyses to what extent countries and
regions have been affected by impacts of weather- related loss events ( storms, floods, heatwaves, etc. ).
The countries and territories affected most in 2018 were Japan, the Philippines as well as Germany. For
the period from 1999 to 2018 Puerto Rico, Myanmar and Haiti rank highest .
• Germanwatch e.V., usually called Germanwatch , is a non -profit , non -governmental organization based in
Bonn , Germany . It also releases Climate Change Performance Index . Hence the correct option is (c )
• United Nations Environment Programme releases the Global Environment Outlook, provides a clear
assessment of the current state of the environment , the challenges that we face and how well we have
dealt with them, with due consideration given to gender, indigenous knowledge and cultural dimensions .
• The Intergovernmental Panel on Climate Change (IPCC) is an intergovernmental body of the
United Nations that is dedicated to providing the world with an objective, scientific information relevant
to understanding the scientific basis of the risk of human -induced climate change, its natural , political , and
economic impacts and risks, and possible response options . Besides the Sixth Assessment Report , to be
completed in 2022 , the IPCC released the Special Report on Global Warming of 1.5 °C in October
2018, released an update to its 2006 Guidelines for National Greenhouse Gas Inventories— the 2019
Refinement— in May 2019

Q 74.C
• The Indian Ocean Commission ( IOC ) is an intergovernmental organization created by the Port-
Louis Declaration in 1982 .
• The IOC was institutionalized in Seychelles in 1984 by the General Agreement for Cooperation , better
known as the Victoria Agreement.
• Composition- IOC is the only regional organization in Africa composed exclusively of islands .
Hence, statement 1 is correct.
• It brings together five-member states
o The Union of Comoros
o Reunion (French overseas territory )
o Madagascar
o Mauritius
o Seychelles. The Maldives is not a member state. Hence , statement 2 is not correct.
• Observer members - China , European Union , Malta , International organization of Francophonie ( is an
international organization representing countries and regions where French is a lingua franca or customary
language ), Japan , United Nations and India . Hence, statement 3 is correct.
• Mandate - To strengthen the ties and to support its member states in the path towards sustainable
development.

Q 75 .C
• UNESCO ' s Man and the Biosphere (MAB) Programme develops the basis within the natural and social
sciences for the rational and sustainable use and conservation of the resources of the biosphere and for the
improvement of the overall relationship between people and their environment .
• Statement 1 is correct: By focusing on sites internationally recognized within the World Network of
Biosphere Reserves, the MAB Programme strives to :
o identify and assess the changes in the biosphere resulting from human and natural activities and
the effects of these changes on humans and the environment, in particular in the context of climate
change;

30 ©Vision IAS
ostudy and compare the dynamic interrelationships between natural/near -natural ecosystems and socio-
economic processes, in particular in the context of accelerated loss of biological and cultural diversity
with unexpected consequences that impact the ability of ecosystems to continue to provide services
critical for human well- being;
o ensure basic human welfare and a liveable environment in the context of rapid urbanization and
energy consumption as drivers of environmental change;
o promote the exchange and transfer of knowledge on environmental problems and solutions, and to
foster environmental education for sustainable development .
• The MAB Strategy 2015 -2025 is an update of the Man and Biosphere ( MAB ) Programme in the new
context of the 2030 Agenda for Sustainable Development and its Sustainable Development Goals
( SDGs ). The Lima Declaration ( 2016 ) on the UNESCO Man and the Biosphere ( MAB) Programme and
its World Network of Biosphere Reserves ( WNBR ) was subscribed by UNESCO Member States,
biosphere reserves, and co -operating public and private sector institutions, as well as civil society
organizations. It expresses their political commitment with the Programme and lays out the principles of
the Lima Action Plan.
• Statement 2 is correct: The Lima Action Plan for UNESCO 's Man and the Biosphere ( MAB )
Programme and its World Network of Biosphere Reserves ( 2016 - 2025 ) contains a comprehensive
set of actions aimed at ensuring the effective implementation of the MAB Strategy 2015-2025.

Q 76 .A
• Soil remediation is a set of processes and techniques of reducing the mobile and in consequence,
bioavailable fraction of contaminants in soils . Under the chemical approaches of soil remediation,
potentially soluble solid phase forms of contaminants are converted to geochemically more stable solid
phases through different soil processes like sorption, ion exchange , and precipitation.
• Different immobilising agents used in chemical approaches are
o Clays and Aluminum Pillared Clays: Due to having high cation exchange capacity and large
specific surface area, clay minerals are potential binding agents for various pollutants. After addition
in soil , acidic aluminum is partially neutralized and is precipitated in the form of its hydroxides at the
surfaces and in the interstitial space of clay minerals . Heavy metals can be adsorbed on a medium
containing clay and Al-hydroxides present on the surface of clay ,

o Zeolites: They are crystalline, hydrated alumino -silicates of alkali and alkaline earth cations . Heavy
metal cations are bonded by the negatively charged sites present at the surfaces of the zeolites and can
also be trapped inside the interconnecting channels and cages ,

o Biochar : It acts as a stabilizing agent in soil as well as adsorbs the metals from the available pool
( soil solution ) or from sewage water.
o Organic Matter: Compost is rich in humic matter and acts as a sinks for trace metals due to having
several binding sites. Therefore it has been recommended as soil amendment to alleviate heavy metal
pollution through a low-cost approach .
o Alkaline Materials: Liming is thought to be the oldest and probably most commonly adopted
immobilizing technique for heavy metal cations in the soil solution and is specif - ically suitable in acid
soils. Lime decreases readily plant available forms ( water-soluble plus exchangeable fraction ) of
metals in soils. Their repeated application at regular intervals is necessary to keep metal immobile.
• Polychlorinated biphenyls (PCBs) are persistent organic pollutants ( POPs ), once widely used in electric
appliances. Many PCBs can still be found in the environment such as in soils and sediments, even though
their use has been heavily restricted . They contaminate environment including soil and water. PCBs
create health problems due to continued presence in soil. Hence option ( a ) is the correct answer .

Q 77.C
• The Paris Agreement was adopted at the Twenty -First Conference of the Parties (COP21) to the United
Nations Framework Convention on Climate Change. A key result of the Paris Agreement negotiations
was the establishment of an enhanced transparency framework for tracking and reporting the progress of
existing and future country commitments, with built- in flexibility included for non -Annex I Parties.
• The COP decision adopting the Paris Agreement decided to establish a Capacity -building Initiative for
44

Transparency in order to build institutional and technical capacity, both pre -and post-2020 ”. The
Capacity -building Initiative for Transparency ( CBIT) was created at the request of Parties to help
strengthen the institutional and technical capacities of non -Annex I countries to meet the enhanced
transparency requirements defined in Article 13 of the Paris Agreement.
• The Capacity -building Initiative for Transparency (CBIT) will support developing country Parties, upon
4

request , in meeting enhanced transparency requirements in a timely manner.


• The CBIT has three aims :
31 ©Vision IAS
o Strengthen national institutions for transparency-related activities in line with national priorities;
o Provide relevant tools, training, and assistance for meeting the provisions stipulated in Article 13 of
the Agreement;
o Assist in the improvement of transparency over time.
• The trust fund for CBIT is administered by the Global Environment Facility (GEF). All developing
country Parties have access to CBIT funds upon request, and all GEF policies and procedures apply to
projects supported by the CBIT.
• Hence both the statements are correct.

Q 78.D
• The Scheduled Tribes and Other Traditional Forest Dwellers (Recognition of Forest Rights) Act, 2006
was enacted to assert the rights of tribal communities over the forestland over which they were
traditionally dependent.
• This Act is crucial to the rights of millions of tribals and other forest dwellers in different parts of our
country as it provides for the restitution of deprived forest rights across India, including both individual
rights to cultivated land in forestland and community rights over common property resources.
• The Act is significant as it provides scope and historic opportunity of integrating conservation and
livelihood rights of the people. Following are the Rights provided under the act:
o right to hold and live in the forest land under the individual or common occupation for
habitation or for self-cultivation for a livelihood by a member or members of a forest-dwelling
Scheduled Tribe or other traditional forest dwellers.
o For the first time, the Forest Rights Act recognises and secures Community Rights or rights over
common property resources of the communities in addition to their individual rights.
o right of ownership, access to collect, use, and dispose of minor forest produce which has been
traditionally collected within or outside village boundaries;
o rights in and over disputed land Rights of settlement and conversion of all forest villages, old
habitation, un-surveyed villages and other villages in forests into revenue villages.
o right to protect, regenerate or conserve or manage any community forest resource which the
communities have been traditionally protecting and conserving for sustainable use.
o rights for the conversion of Pattas or leases or grants issued by any local authority or any State
Government on forest lands to titles;
o right of access to biodiversity and community right to intellectual property and traditional
knowledge related to biodiversity and cultural diversity;
o right to in situ rehabilitation including alternative land in cases where the Scheduled Tribes or other
traditional forest dwellers have been illegally evicted or displaced from forest land of any description
without receiving their legal entitlement to rehabilitation.
• Hence, option (d) is the correct answer.

Q 79.C
• Convention on the Conservation of Migratory Species of Wild Animals (CMS) is an environmental
treaty under the aegis of the United Nations Environment Programme (UNEP), CMS provides a
global platform for the conservation and sustainable use of migratory animals and their habitats.
• As the only global convention specializing in the conservation of migratory species, their habitats
and migration routes, CMS complements and co-operates with a number of other international
organizations, NGOs and partners in the media as well as in the corporate sector.
• CMS brings together the States through which migratory animals pass, the Range States, and lays the
legal foundation for internationally coordinated conservation measures throughout a migratory range.
• Migratory species threatened with extinction are listed on Appendix I of the Convention. CMS Parties
strive towards strictly protecting these animals, conserving or restoring the places where they live,
mitigating obstacles to migration and controlling other factors that might endanger them.
• Hence both statements 1 and 2 are correct.

Q 80.B
• Recent Context - Electronic Vaccine Intelligence Network (eVIN) has ensured essential immunization
services during the COVID pandemic.
32 www.visionias.in ©Vision IAS
• About e-VIN
o It is an innovative technological solution aimed at strengthening immunization supply chain systems
across the country.
o It aims to provide real-time information on vaccine stocks and flows, and storage temperatures across
all cold chain points in the country.
o It enables real-time monitoring of stock and storage temperature of the vaccines kept in multiple
locations across the country.
o It has been developed by the Ministry of Health and Family Welfare, Government of India.
Hence statement 1 is not correct.
o It has been developed to support the Government of India’s Universal Immunization program
by providing real-time information on vaccine stocks and flows.
o This is being implemented under the National Health Mission (NHM) by the Ministry of Health
and Family Welfare. Hence statement 2 is correct.
o This system has been used during the COVID pandemic for ensuring the continuation of the essential
immunization services and protecting our children and pregnant mothers against vaccine-preventable
diseases.
• Benefits of e-VIN
o It has reached 32 States and Union Territories (UTs) and will soon be rolled-out in the remaining
States and UTs of Andaman & Nicobar Islands, Chandigarh, Ladakh, and Sikkim.
o It has helped create a big data architecture that generates actionable analytics encouraging data-driven
decision-making and consumption-based planning that helps in maintaining optimum stocks of
vaccines leading to cost savings.
o Vaccine availability at all times has increased to 99% in most health centers. While instances of stock-
outs have reduced by 80%, the time taken to replenish stocks has also decreased by more than half, on
an average. This has ensured that every child who reaches the immunization session site is
immunized, and not turned back due to the unavailability of vaccines.

Q 81.C
• The National Clean Air Programme is a pollution control initiative that was launched by the Ministry of
Environment with the intention to cut the concentration of coarse (particulate matter of diameter 10
micrometer or less, or PM10) and fine particles (particulate matter of diameter 2.5 micrometer or less,
or PM2.5) by at least 20% by 2024, with 2017 as the base year for comparison. Hence statement 1 is
correct.
• The approach for NCAP includes collaborative, multi-scale and cross-sectoral coordination between the
relevant central ministries, state governments and local bodies.
• There will be use of the Smart Cities program to launch the NCAP in the 43 smart cities falling in the
list of the 102 non-attainment cities for which city specific plans are being formulated.
• Other features of NCAP include, increasing number of monitoring stations in the country including rural
monitoring stations, technology support, emphasis on awareness and capacity building initiatives,
setting up of certification agencies for monitoring equipment, source apportionment studies, emphasis on
enforcement, specific sectoral interventions etc. Hence statement 2 is correct.

Q 82.A
• Sulfur oxides (SOx) are compounds of sulfur and oxygen molecules. Sulfur dioxide (SO2) is the
predominant form found in the lower atmosphere. It is a colorless gas that can be detected by taste and
smell.
• Major Sources:
o Anthropogenic sources: Most sulfur dioxide is produced by burning fuels containing sulfur or by
roasting metal sulfide ores. Other anthropogenic sources include the burning of coal, use
of petroleum products and industrial production of sulphuric acid in metallurgical, chemical
and fertiliser industries.
o Natural sources: The natural sources of sulfur dioxide include volcanic eruptions and Biological
process such as decomposition in soil. Hence statement 1 is correct.
• Impacts on health and Environment:
o Sulfur dioxide dissolves readily in water present in the atmosphere to form sulfurous acid (H2SO3).
About 30% of the sulfur dioxide in the atmosphere is converted to sulfate aerosol (acid aerosol),

33 www.visionias.in ©Vision IAS


which is removed through wet or dry deposition processes which is known as acid rain. Sulfur
trioxide (SO3), another oxide of sulfur, is either emitted directly into the atmosphere or produced
from sulfur dioxide and is rapidly converted to sulfuric acid (H2SO4).
o Sulfur dioxide and other SOx are partly culpable in the formation of thick haze and smog, which can
impair visibility in addition to impacting health. Hence statement 2 is correct.
o Sulfur dioxide affects the respiratory system, particularly lung function, and can irritate the eyes.
o Sulfur dioxide irritates the respiratory tract and increases the risk of tract infections. It causes
coughing, mucus secretion and aggravates conditions such as asthma and chronic bronchitis.
• National Air Quality Index was launched in 2015 to disseminate air quality information. AQI considers
eight pollutants (Particulate Matter (PM10), Particulate Matter (PM2. 5), Nitrogen Dioxide
(NO2), Sulphur Dioxide (SO2), Carbon Monoxide (CO), Ozone (O3), Ammonia (NH3), and Lead (Pb)
for which National Ambient Air Quality Standards are prescribed in major cities. Hence statement 3 is
correct.
• Lichens are mutualistic associations of a fungus and an alga and occur as crusty patches or bushy
growths on trees, rocks and bare ground. Lichens are widely used as environmental indicators. Lichens
are sensitive to atmospheric pollution such as sulphur oxides because they receive all their nutrients and
water from wet and dry atmospheric deposition. Increased levels of SO 2 in the air injure lichens. When
the air is very badly polluted with sulphur dioxide there may be no lichens present, just green algae
may be found. If the air is clean, shrubby, hairy and leafy lichens become abundant.

Q 83.C
• The conservation and development of forest primarily involve three strategies – afforestation through
natural/artificial regeneration, protection and management. MoEFCC is implementing the National
Afforestation Programme (NAP) for afforestation of degraded forest lands.
• National Afforestation Programme is a major afforestation scheme of the National Afforestation
and Eco-Development Board (NAEB) of the Government. It continues to be the flagship scheme of
NAEB.
• The overall objective of the National Afforestation Programme (NAP) scheme is ecological
restoration of degraded forests and to develop the forest resources with peoples’ participation, with
focus on improvement in livelihoods of the forest-fringe communities, especially the poor.
• NAP aims to support and accelerate the on-going process of devolving forest conservation, protection,
management and development functions to the Joint Forest Management Committees (JFMCs) at the
village level, which are registered societies.
• NAP Scheme aims to support and accelerate the ongoing process of devolving forest protection,
management and development functions to decentralized institutions of Joint Forest Management
Committee (JFMC) at the village level, and Forest Development Agency (FDA) at the forest division
level.
• The village is reckoned as a unit of planning and implementation and all activities under the
programme are conceptualized at the village level.
• The National Afforestation and Eco-Development Board (NAEB), was set up in August 1992. It is
responsible for promoting afforestation, tree planting, ecological restoration and eco-development
activities in the country, with special attention to the degraded forest areas and lands adjoining the forest
areas, national parks, sanctuaries and other protected areas as well as the ecologically fragile areas like the
Western Himalayas, Aravallis, Western Ghats, etc.
• Hence option (c) is the correct answer.

Q 84.A
• The Central government issued Coastal Regulation Zone (CRZ) Notification, 2018 which was last
reviewed and issued in 2011, with periodic amendments to some clauses. It was issued under the
provisions of the Environment (Protection) Act, 1986. Hence statement 1 is correct.
• Salient Features:
o As per CRZ - 2011 Notification, for CRZ-II areas, Floor Space Index (FSI) or the Floor Area Ratio
(FAR) had been frozen at 1991 Development Control Regulation (DCR) levels. In the CRZ, 2018
Notification, it has been proposed to de-freeze the same and permits FSI for construction
projects, as prevailing on the date of the new Notification. Hence statement 3 is not correct.
o Densely populated rural areas to be afforded greater opportunity for development: CRZ-III A zones
which are densely populated rural areas shall have a No Development Zone (NDZ) of 50
meters from the HTL. CRZ-III B zones which are Rural areas with low population density shall
continue to have an NDZ of 200 meters from the HTL.

34 www.visionias.in ©Vision IAS


o Temporary tourism facilities such as shacks, toilet blocks, change rooms, drinking water facilities
etc. have now been permitted in Beaches and in the 'No Development Zone' of CRZ-III
areas. The procedure for CRZ clearances has been streamlined.
o For islands close to the main land coast and for all Backwater Islands in the main land, No
Development Zone of 20 m has been stipulated. Hence statement 2 is not correct.
o Specific guidelines related to their conservation and management plans have been drawn up as a part
of the CRZ Notification.
o Defence and strategic projects have been accorded necessary dispensation.
• Hence option (a) is the correct answer.

Q 85.B
• In a positive move towards fighting climate change, 197 countries agreed upon an amendment to the
Montreal Protocol for phasing down the planet-warming hydrofluorocarbon (HFC) gases. While HFCs
do not deplete the ozone layer substantially, their global warming potential is thousand times more
than carbon dioxide.
• Under the agreement, developed countries will reduce HFC emissions use first, followed by a group of
Article 5 countries including China. India and nine other countries in South and West Asia will follow
suit. Overall, the agreement is expected to reduce HFC use by 85 per cent by 2045. Hence, statement
1 is not correct.
• Countries are divided into three groups, as per their phase-down schedules to freeze and reduce the
production of HFCs. The developed countries, led by the US and Europe, will reduce HFC use by 85 per
cent by 2036 over a 2011-13 baseline. China, which is the largest producer of HFCs in the world, will
reduce HFC use by 80 per cent by 2045 over the 2020-22 baseline. India will reduce the use of HFCs by
85 per cent over the 2024-26 baseline. The amendment also increases funding support to developing
countries.
• In addition to their phase-down deadlines, developed nations have agreed to cut 70 per cent of their HFC
use by 2029. This means India will start reducing its HFC consumption when the developed countries
would have reduced their consumption by 70 per cent.
• The Agreement upholds the principle of Common but Differentiated Responsibilities and
Respective Capabilities. The agreement recognises the development imperatives of high-growth
economies like India and provides a realistic and viable roadmap for the implementation of a phase-out
schedule.
• The Kigali Amendment to the Montreal Protocol came into force on 1 January 2019. The threshold
for the agreement to enter into force was met on 17 November 2017, when it was ratified by 20 parties.
• Hence statements 2 and 3 are correct.

Q 86.C
• The Ministry of Environment, Forest and Climate Change (MoEFCC) has developed the criteria of
categorization of industrial sectors based on the Pollution Index which is a function of the emissions (air
pollutants), effluents (water pollutants), hazardous wastes generated and consumption of resources.
The purpose of the categorization is to ensure that the industry is established in a manner which is
consistent with the environmental objectives and prompt industrial sectors willing to adopt cleaner
technologies, ultimately resulting in the generation of fewer pollutants.
• The Pollution Index of any industrial sector is a number from 0 to 100 and the increasing value of PI
denotes the increasing degree of pollution load from the industrial sector.
• Red Category: It refers to industrial Sectors having Pollution Index score of 60 and above. The list
includes Lubricating oils and grease manufacturing, Power generation units, Sugar industry, E-waste
recyclers, Milk and dairy products, Health care establishment, Ship breaking etc.
• Orange Category: It refers to Industrial Sectors having Pollution Index score of 41 to 59. It includes
Aluminium and Copper extraction from scrap, Ayurvedic and Homeopathic medicine, Brickfields, Coal
washeries, Dry cell battery, Hot mix plants etc.
• Green category: It refers to Industrial Sectors having Pollution Index score of 21 to 40. The list
includes Aluminium utensils, Biomass briquettes, Dal mills, Leather foot wear, Blending of lubricating
oil etc.

35 www.visionias.in ©Vision IAS


• White category: It includes Industrial Sectors having Pollution Index score incl.& upto 20. It
includes Organic manure, Surgical and medical products assembly, Chalk making, Coir items from
coconut husks, Making fly ash bricks, Glass ampules, Solar module non-conventional energy apparatus
etc. Hence option (c) is the correct answer.

Q 87.D
• Recent Context - The Commission for Air Quality Management in National Capital Region and
Adjoining Areas Ordinance, 2020 was promulgated on October 28, 2020. It provides for the constitution
of the Commission for Air Quality Management in the National Capital Region and Adjoining regions. It
also dissolves Environment Pollution (Prevention and Control) Authority (EPCA) for the NCR.
• About the commission:
o The commission has been constituted for better co-ordination, research, identification, and resolution
of problems related to air quality in the national capital region (NCR) and adjoining areas.
• Composition of the New Commission:
o The Commission will be headed by a full-time chairperson who has been a Secretary to the
Government of India, or a Chief Secretary to a State government. Hence statement 1 is not
correct.
o The chairperson will hold the post for three years or until s/he attains the age of 70 years.
o It will have members from several Ministries as well as representatives from the stakeholder
States. Hence statement 2 is not correct.
o It will have experts from the CPCB, the Indian Space Research Organisation (ISRO), and civil society
institutions.
• Powers
o In matters of air pollution and air quality management, the Commission will supersede all existing
bodies such as the CPCB, and even the state governments of Haryana, Punjab, Rajasthan, and Uttar
Pradesh. It will have the powers to issue directions to the states.
▪ CPCB and its State branches have the powers to implement provisions of the Environment
Protection Act, 1986 for air, water and land pollution. Their powers continue. However, in case of
dispute or a clash of jurisdictions, the Commission’s writ will prevail specifically to matters
concerning air pollution.
o The Commission will also coordinate efforts of state governments to curb air pollution, and will lay
down the parameters of air quality for the region.
o It will have powers to restrict the setting up of industries in vulnerable areas and will be able to
conduct site inspections of industrial units.
o If its directions are contravened, the Commission will have the power to impose a fine of up to Rs. 1
crore and imprisonment of up to 5 years.
o Only the National Green Tribunal (NGT), and not civil courts, is authorized to hear cases where the
Commission is involved.

Q 88.B
• The Global Tiger Initiative (GTI) was launched in 2008 as a global alliance of governments,
international organizations, civil society, the conservation and scientific communities and the private
sector, with the aim of working together to save wild tigers from extinction. In 2013, the scope was
broadened to include Snow Leopards. The GTI’s founding partners included the World Bank, the
Global Environment Facility (GEF), the Smithsonian Institution, Save the Tiger Fund, and
International Tiger Coalition (representing more than 40 non-government organizations). The
initiative is led by the 13 tiger range countries (TRCs). Global Tiger Recovery Programme was
endorsed by the Tiger Range Countries (TRCs) in the first “Tiger Summit” in St Petersburg, Russia in
November 2010. Hence option (b) is the correct answer.
• Global Tiger Forum is an inter-governmental international body working exclusively for the
conservation of Tigers in the wild utilizing co-operative policies, common approaches, technical
expertise, scientific modules and other appropriate programmes. The GTF was formed in 1993 on
recommendations from an international symposium on Tiger Conservation at New Delhi, India. The
first meeting of the Tiger Range countries to set up the forum was held in 1994, in which India was
elected to the Chair and was asked to form an interim secretariat. In 1997, the GTF became an
independent organization. The Global Tiger Forum (GTF) is the only inter-governmental international

36 www.visionias.in ©Vision IAS


body established with members from willing countries to embark on a global campaign to protect the
Tiger.
• The International Tiger Coalition is an alliance of many organizations representing more than 40
organizations across the globe, united under the common aim of stopping trade in tiger parts and products
from all sources.

Q 89.B
• Thermal pollution is defined as a sudden increase or decrease in temperature of a natural body of water.
This normally occurs when a plant or a facility (eg. thermal plants and nuclear plants) takes in water from
a natural resource and puts it back with an altered temperature.
• Effects on aquatic systems when warm water is released:
o The warm temperature reduces the levels of DO (Dissolved Oxygen) in water. The warm water
holds relatively less oxygen than cold water. The decrease in DO can create suffocation for plants and
animals such as fish, amphibians etc. Warmer water allows algae to flourish which can decrease
oxygen levels in the water. Hence statement 2 is correct.
o Increase in Toxins: With the constant flow of high-temperature discharge from industries, there is a
huge increase in toxins that are being regurgitated into the natural body of water.
o Loss of Biodiversity: Changes in the environment may cause certain species of organisms to shift
their base to some other place while there could be a significant number of species that may shift in
because of warmer waters.
o Ecological Impact: A sudden thermal shock can result in mass killings of fish, insects, plants or
amphibians. Hotter water may prove favorable for some species, while it could be lethal for other
species.
o Affects Reproductive Systems: Excessive temperature can cause the release of immature eggs or can
prevent the normal development of certain eggs.
o Increases Metabolic Rate: Thermal pollution increases the metabolic rate of organisms as
increasing enzyme activity occurs that causes organisms to consume more food. It disrupts the
stability of the food chain and alters the balance of species composition. Hence statement 1 is not
correct.
• Solutions to thermal pollution includes cooling ponds, Artificial lakes, water recycling etc.

Q 90.B
• Particulate matter is the sum of all solid and liquid particles suspended in air many of which are
hazardous. This complex mixture includes both organic and inorganic particles, such as dust, pollen, soot,
smoke, and liquid droplets.
• Based on size, particulate matter is often divided into two main groups:
o The coarse fraction contains the larger particles with a size ranging from 2.5 to 10 µm (micrometers)
(PM10 - PM2.5). Hence statement 1 is not correct.
o The fine fraction contains the smaller ones with a size up to 2.5 µm (PM2.5). The particles in the fine
fraction which are smaller than 0.1 µm are called ultrafine particles.
• Particulate matter particles are emitted directly from a source, such as construction sites, unpaved roads,
fields, smokestacks or fires. Particulate matter cause serious health problems.
• Dioxins are a group of chemically-related compounds that are persistent environmental pollutants
(POPs). Dioxins are unwanted by-products of a wide range of manufacturing processes including
smelting, chlorine bleaching of paper pulp and the manufacturing of some herbicides and
pesticides. Hence statement 2 is correct.
• They accumulate in food chain, mainly in the fatty tissues. Exposure to dioxins may result in skin lesions,
altered liver function, impairment of the immune system, the endocrine system and reproductive
functions.
• Acid rain is a broad term that includes any form of precipitation with acidic components, such as sulfuric
or nitric acid that fall to the ground from the atmosphere in wet or dry forms. This can include rain, snow,
fog, hail or even dust that is acidic. Acid rain results when sulfur dioxide (SO2) and nitrogen oxides
(NOX) are emitted into the atmosphere and transported by wind and air currents. The SO2 and NOX react
with water, oxygen and other chemicals to form sulfuric and nitric acids. These then mix with water and
other materials before falling to the ground. Hence statement 3 is correct.

37 www.visionias.in ©Vision IAS


• The effects of acid rain leave trees and plants less healthy, more vulnerable to cold temperatures, insects,
and disease. The pollutants may also inhibit trees ability to reproduce. Acid deposits damage physical
structures such as limestone buildings and cars.

Q 91.C
• The ‘Biodiversity Heritage Sites’ (BHS) are unique ecosystems having rich biodiversity comprising of
any one or more of the following components:
o The richness of wild as well as domesticated species or intra-specific categories.
o High endemism.
o Presence of rare and threatened species, keystone species, species of evolutionary significance.
o Wild ancestors of domestic/cultivated species or their varieties.
o Past pre-eminence of biological components represented by fossil beds and having significant
cultural, ethical or aesthetic values and are
o important for the maintenance of cultural diversity, with or without a long history of human
association with them.
• Under Section 37 of Biological Diversity Act, 2002 (BDA) the State Government in consultation with
local bodies may notify in the official gazette, areas of biodiversity importance as Biodiversity
Heritage Sites (BHS).
• Further, the State Government in consultation with the Central Government may frame rules for the
management and conservation of BHS.
• The creation of BHS may not put any restriction on the prevailing practices and usages of the local
communities, other than those voluntarily decided by them. The purpose of declaring BHS is to enhance
the quality of life of the local communities through conservation of such sites.
• Hence, both statements are correct.

Q 92.B
• Biofuels are energy sources made from recently grown biomass (plant or animal matter). Biofuels
have been around for a long time, but petroleum and coal have been used primarily as energy sources due
to their high abundance, high energy value, and cheap prices.
• There are three types of biofuels: 1st, 2nd and 3rd generation biofuels. They are characterized by their
sources of biomass, their limitations as a renewable source of energy, and their technological progress.
• The main drawback of 1st generation biofuels is that they come from biomass that is also a food source.
This presents a problem when there is not enough food to feed everyone.
• 2nd generation biofuels come from non-food biomass but still compete with food production for
land use. Finally, 3rd generation biofuels present the best possibility for alternative fuel because they
don’t compete with food. However, there are still some challenges in making them economically
feasible.
• Third generation biofuels are more energy-dense than 1st and 2nd generation biofuels per area of
harvest. They are cultured as low-cost, high-energy, and completely renewable sources of energy.
• Algae are advantageous in that it can grow in areas unsuitable for 1st and 2nd generation crops, which
would relieve stress on water and arable land used. It can be grown using sewage, wastewater, and
saltwater, such as oceans or salt lakes. Because of this, there wouldn't be a need to use water that would
otherwise be used for human consumption. The algae can be converted into various types of renewable
biofuels including bioethanol, biodiesel, biogas, photobiologically produced biohydrogen, and further
processing for bio-oil and syngas production through liquefaction and gasification, respectively.
• A further benefit of algae based biofuels is that the fuel can be manufactured into a wide range of
fuels such as diesel, petrol and jet fuel.
• However, further research still needs to be done to further the extraction process in order to make it
financially competitive to petrodiesel and other petroleum-based fuels. Hence the correct answer is
option (b).
Q 93.C
• Strategic Approach to International Chemicals Management (SAICM) a part of the United Nations
Environment Programme was adopted by the First International Conference on Chemicals
Management (ICCM1) on 6 February 2006 in Dubai, Strategic Approach to International Chemicals
Management (SAICM) is a policy framework to promote chemical safety around the world. Hence
statement 1 is correct and statement 2 is not correct.
• SAICM was developed by a multi-stakeholder and multi-sectoral Preparatory Committee and
supports the achievement of the 2020 goal agreed upon at the 2002 Johannesburg World Summit on
Sustainable Development.
38 www.visionias.in ©Vision IAS
• SAICM's overall objective is the achievement of the sound management of chemicals throughout their
life cycle so that by the year 2020, chemicals are produced and used in ways that minimize significant
adverse impacts on the environment and human health.
• The 'lead-in Paint' component is part of the SAICM Global Environment Facility (GEF) project
“Global best practices on emerging chemical policy issues of concern under the Strategic Approach
to International Chemicals Management". The two other components are chemicals in products and
knowledge management. Hence statement 3 is correct.
• The project outcome is for 40 countries including India to legislate and implement legislation to restrict
the use of lead paint and for at least 50 small and medium enterprises (SME) paint manufacturers in eight
countries to phase out lead from their production processes.
• India initiated the preparation of the National Chemicals Management Profile to assess India’s
infrastructure and capacity for the management of chemicals. Hence option (c) is the correct
answer.

Q 94.B
• Recent Context – United Nations has recently launched ‘The Sanitation and Hygiene Fund’. Hence
option (b) is the correct answer.
• About the Fund:
o The fund aims to provide accelerated funding to countries with the heaviest burden of diseases
stemming from lack of sanitation services and have the least ability to respond to them. It also aims to
raise $2 billion over the next five years for these countries.
o The fund is hosted by the UN Office for Project Services, which provides technical advice and
project implementation to the UN and its partners.
• The objectives of the Fund are:
o Expanding household sanitation.
o Ensuring menstrual health and hygiene.
o Providing sanitation and hygiene in schools and healthcare facilities.
o Supporting innovative sanitation solutions.
• The fund will support countries to achieve Sustainable Development Goal (SDG) 6.2 through
sustainable, country-led approaches.
• Sanitation, hygiene, and menstrual health improve people's lives. Safe Sanitation and hygiene are also
central to containing the rapid spread of the COVID-19 pandemic.
• About SDG 6.2 - By 2030, achieve access to adequate and equitable sanitation and hygiene for all and
end open defecation, paying special attention to the needs of women and girls and those in vulnerable
situations.

Q 95.C
• The term ‘Greenhouse effect’ has been derived from a phenomenon that occurs in a greenhouse. It looks
like a small glasshouse and is used for growing plants especially during winter.
• In a greenhouse, the glass panel lets the light in but does not allow heat to escape. Therefore, the
greenhouse warms up, very much like inside a car that has been parked in the sun for a few hours.
• The greenhouse effect is a naturally occurring phenomenon that is responsible for the heating of
Earth’s surface and atmosphere, without the greenhouse effect the average temperature at the surface of
Earth would have been a chilly –18 degree C rather than the present average of 15 degrees C.
• Clouds and gases reflect about one-fourth of the incoming solar radiation and absorb some of it but
almost half of the incoming solar radiation falls on Earth’s surface heating it, while a small proportion is
reflected back.
• Earth’s surface re-emits heat in the form of infrared radiation but part of this does not escape into
space as atmospheric gases (e.g., carbon dioxide, methane, etc.) absorb a major fraction of it.
• The molecules of these gases radiate heat energy, and a major part of which again comes to Earth’s
surface, thus heating it up once again. This cycle is repeated many times.
• The above-mentioned gases – carbon dioxide and methane – are commonly known as greenhouse gases
because they are responsible for the greenhouse effect. The relative contribution of various greenhouse

39 www.visionias.in ©Vision IAS


gases to total global warming in increasing order is nitrous oxide (6%), CFCs (14%), Methane
(20%), Carbon dioxide (60%). Hence the correct answer is option (c).
• An increase in the level of greenhouse gases has led to considerable heating of Earth leading to global
warming. During the past century, the temperature of Earth has increased by 0.6 degrees C, most of it
during the last three decades.

Q 96.A
• In partnership with the United Nations Environment Programme, the Ocean Agency has launched
Glowing Glowing Gone, a creative awareness campaign that draws attention towards conservation of
coral reefs.
• Coral fluorescence, or “glowing” coral, is a last line of defence before coral dies and bleaches. In a
desperate attempt to survive increasing ocean heat waves due to climate change, some corals glow in
vibrant colour. The corals produce brightly coloured chemicals in their flesh that act as a sunscreen.
• Through Glowing Glowing Gone, The Ocean Agency hopes to garner public support to inspire policy and
funding to conserve coral reefs and save an ecosystem on which our entire planet depends.
• Hence option (a) is the correct answer.

Q 97.D
• A catalytic converter is an exhaust emission control device that reduces toxic gases and pollutants in
exhaust gas from an internal combustion engine into less-toxic pollutants by catalyzing a redox reaction
(an oxidation and a reduction reaction). Catalytic converters are usually used with internal combustion
engines fueled by either gasoline or diesel.
• Automobile engines emit a lot of harmful substances. Some of them are carbon dioxide (CO2), nitrogen
oxides (NOx), sulphur oxides and volatile organic compounds.
• On a car, the catalytic converter is attached to the exhaust pipe. A metal casing contains a ceramic
honeycomb. The honeycomb is coated with a mix of platinum (Pt), palladium (Pd) and rhodium
(Rh). These noble metals are good at resisting oxidation, corrosion and acid.
• The noble metals in catalytic converters act as catalysts. The honeycomb structure inside a catalytic
converter maximizes the surface area where reactions can take place.
• Catalytic converters use reduction and oxidation (redox) reactions to reduce harmful emissions. The
reduction reaction reduces nitrogen oxides (NOx) by removing nitrogen atoms from nitrogen oxide
molecules (NO and NO2). This reaction creates free oxygen and nitrogen gases. Oxidation reaction
involves combination of unburnt hydrocarbons and oxygen to form carbon dioxide and water.
• Catalytic converters requires the use unleaded fuel, because the lead in conventional fuel can coat the
surface that normally reacts with the exhaust gases and thus decreases the efficiency.
• Hence option (d) is the correct answer.

Q 98.C
• Biomethanation is a process by which organic material is microbiologically converted under
anaerobic conditions to biogas. Three main physiological groups of microorganisms are involved:
fermenting bacteria, organic acid oxidizing bacteria, and methanogenic archaea.
• Methanogenesis or biomethanation is the formation of methane by microbes known as methanogens.
Organisms capable of producing methane have been identified only from the domain Archaea, a group
phylogenetically distinct from both eukaryotes and bacteria, although many live in close association with
anaerobic bacteria.

Q 99.C
• The Compensatory Afforestation Fund Act, 2016 was enacted to manage the funds collected for
compensatory afforestation which till then was managed by ad hoc Compensatory Afforestation Fund
Management and Planning Authority (CAMPA).
• Highlights of the to The Compensatory Afforestation Fund Act, 2016:
o It establishes the National Compensatory Afforestation Fund under the Public Account of
India, and a State Compensatory Afforestation Fund under the Public Account of each state.
Hence, statement 1 is correct.

40 www.visionias.in ©Vision IAS


o These Funds will receive payments for: (i) compensatory afforestation, (ii) net present value of forest
(NPV), and (iii) other project specific payments. The National Fund will receive 10% of these
funds, and the State Funds will receive the remaining 90%. Hence, statements 2 not correct.
o These Funds will be primarily spent on afforestation to compensate for loss of forest cover,
regeneration of forest ecosystem, wildlife protection and infrastructure development.
o It also establishes the National and State Compensatory Afforestation Fund Management and
Planning Authorities to manage the National and State Funds. Hence, statements 3 is correct.

Q 100.B
• The indiscriminate disposal of plastic has become a major threat to the environment. Every year, millions
of plastic bags end up in to the environment vis-a-vis soil, water bodies, water courses, etc. Therefore, to
address the issue of scientific plastic waste management, the Government has notified the Plastic Waste
Management Rules, 2016, in suppression of the earlier Plastic Waste (Management and Handling) Rules,
2011.
• Important Provisions:
o The Plastic Waste Management Rules, 2016 aim to Increase minimum thickness of plastic carry
bags from 40 to 50 microns. Hence statement 1 is not correct.
o The rules expand the jurisdiction of applicability from the municipal area to rural areas, because
plastic has reached rural areas also.
o It introduce collection of plastic waste management fee through pre-registration of the producers,
importers of plastic carry bags/multilayered packaging and vendors selling the same for establishing
the waste management system.
o Extended Producer Responsibility: The producers have been made responsible for collecting
waste generated from their products. They have to approach local bodies for formulation of
plan/system for the plastic waste management within the prescribed timeframe. State Pollution
Control Board (SPCBs) will not grant unless the producer proposes the action plan. Hence
statement 2 is correct.
o Central Pollution Control Board (CPCB) has been mandated to formulate the guidelines for thermoset
plastic (plastic difficult to recycle).
o It promotes use of plastic waste for road construction as per Indian Road Congress guidelines.
• The rules were amended in 2018 to provide phasing out of Multilayered Plastics which are non-
recyclable or non-energy reconverable, or with no alternate use.

41 www.visionias.in ©Vision IAS

You might also like